Expected Number of Coin Tosses to Get Five Consecutive Heads












56












$begingroup$


A fair coin is tossed repeatedly until 5 consecutive heads occurs.



What is the expected number of coin tosses?










share|cite|improve this question











$endgroup$








  • 4




    $begingroup$
    Yet another copy and paste from Brilliant.org: brilliant.org/i/5rCgJ3
    $endgroup$
    – Erick Wong
    Apr 17 '13 at 5:49






  • 2




    $begingroup$
    @ErickWong: Is this a recent problem on brilliant.org?
    $endgroup$
    – robjohn
    Apr 17 '13 at 8:48
















56












$begingroup$


A fair coin is tossed repeatedly until 5 consecutive heads occurs.



What is the expected number of coin tosses?










share|cite|improve this question











$endgroup$








  • 4




    $begingroup$
    Yet another copy and paste from Brilliant.org: brilliant.org/i/5rCgJ3
    $endgroup$
    – Erick Wong
    Apr 17 '13 at 5:49






  • 2




    $begingroup$
    @ErickWong: Is this a recent problem on brilliant.org?
    $endgroup$
    – robjohn
    Apr 17 '13 at 8:48














56












56








56


70



$begingroup$


A fair coin is tossed repeatedly until 5 consecutive heads occurs.



What is the expected number of coin tosses?










share|cite|improve this question











$endgroup$




A fair coin is tossed repeatedly until 5 consecutive heads occurs.



What is the expected number of coin tosses?







probability contest-math






share|cite|improve this question















share|cite|improve this question













share|cite|improve this question




share|cite|improve this question








edited Dec 2 '15 at 22:33









Daniel Fischer

174k17169288




174k17169288










asked Apr 17 '13 at 3:48









leava_sinusleava_sinus

316154




316154








  • 4




    $begingroup$
    Yet another copy and paste from Brilliant.org: brilliant.org/i/5rCgJ3
    $endgroup$
    – Erick Wong
    Apr 17 '13 at 5:49






  • 2




    $begingroup$
    @ErickWong: Is this a recent problem on brilliant.org?
    $endgroup$
    – robjohn
    Apr 17 '13 at 8:48














  • 4




    $begingroup$
    Yet another copy and paste from Brilliant.org: brilliant.org/i/5rCgJ3
    $endgroup$
    – Erick Wong
    Apr 17 '13 at 5:49






  • 2




    $begingroup$
    @ErickWong: Is this a recent problem on brilliant.org?
    $endgroup$
    – robjohn
    Apr 17 '13 at 8:48








4




4




$begingroup$
Yet another copy and paste from Brilliant.org: brilliant.org/i/5rCgJ3
$endgroup$
– Erick Wong
Apr 17 '13 at 5:49




$begingroup$
Yet another copy and paste from Brilliant.org: brilliant.org/i/5rCgJ3
$endgroup$
– Erick Wong
Apr 17 '13 at 5:49




2




2




$begingroup$
@ErickWong: Is this a recent problem on brilliant.org?
$endgroup$
– robjohn
Apr 17 '13 at 8:48




$begingroup$
@ErickWong: Is this a recent problem on brilliant.org?
$endgroup$
– robjohn
Apr 17 '13 at 8:48










10 Answers
10






active

oldest

votes


















77












$begingroup$

Let $e$ be the expected number of tosses. It is clear that $e$ is finite.



Start tossing. If we get a tail immediately (probability $frac{1}{2}$) then the expected number is $e+1$. If we get a head then a tail (probability $frac{1}{4}$), then the expected number is $e+2$. Continue $dots$. If we get $4$ heads then a tail, the expected number is $e+5$. Finally, if our first $5$ tosses are heads, then the expected number is $5$. Thus
$$e=frac{1}{2}(e+1)+frac{1}{4}(e+2)+frac{1}{8}(e+3)+frac{1}{16}(e+4)+frac{1}{32}(e+5)+frac{1}{32}(5).$$
Solve this linear equation for $e$. We get $e=62$.






share|cite|improve this answer









$endgroup$









  • 1




    $begingroup$
    It is clear that e is finite, but how can you show it properly though ? Thanks.
    $endgroup$
    – Dark
    Jul 3 '15 at 17:39






  • 2




    $begingroup$
    If one wants, let $X$ be the number of tosses. Then $Pr(X=n)le (1/2)^{n-5}$. So $E(X)le sum n (1/2)^{n-5}$, a convergent series.
    $endgroup$
    – André Nicolas
    Jul 3 '15 at 17:58








  • 14




    $begingroup$
    The same method obviously generalizes to give $e_n$, the expected number of tosses to get $n$ consecutive heads ($n ge 1$): $$e_n=frac{1}{2}(e_n+1)+frac{1}{4}(e_n+2)+frac{1}{8}(e_n+3)+frac{1}{16}(e_n+4)+cdots +frac{1}{2^n}(e_n+n)+frac{1}{2^n}(n),$$ the solution of which is easily found to be $$e_n = 2(2^n - 1).$$
    $endgroup$
    – r.e.s.
    Jul 19 '15 at 23:57






  • 4




    $begingroup$
    Why are TT, TTT not considered?
    $endgroup$
    – Jaydev
    Jul 24 '17 at 1:24






  • 4




    $begingroup$
    @Jaydev TT and TTT both are covered by the case "if we get a tail immediately".
    $endgroup$
    – David K
    Oct 6 '17 at 13:13



















21












$begingroup$

Here is a generating function approach.



Consider the following toss strings, probabilities, and terms



$$
color{#00A000}{
begin{array}{llc}
T&frac12&qquadfrac12x\
HT&frac14&qquadfrac14x^2\
HHT&frac18&qquadfrac18x^3\
HHHT&frac1{16}&qquadfrac1{16}x^4\
HHHHT&frac1{32}&qquadfrac1{32}x^5\
color{#C00000}{HHHHH}&color{#C00000}{frac1{32}}&color{#C00000}{qquadfrac1{32}x^5}
end{array}
}
$$
Each term has the probability as its coefficient and the length of the string as its exponent.



Possible outcomes are any combination of the green strings followed by the red string. We get the generating function of the probability of ending after $n$ tosses to be
$$
begin{align}
f(x)&=sum_{k=0}^inftyleft(frac12x+frac14x^2+frac18x^3+frac1{16}x^4+frac1{32}x^5right)^kfrac1{32}x^5\
&=frac{frac1{32}x^5}{1-left(frac12x+frac14x^2+frac18x^3+frac1{16}x^4+frac1{32}x^5right)}\
&=frac{frac1{32}x^5}{1-frac{frac12x-frac1{64}x^6}{1-frac12x}}\
&=frac{frac1{32}x^5-frac1{64}x^6}{1-x+frac1{64}x^6}
end{align}
$$
The average duration is then
$$
begin{align}
f'(1)
&=left.frac{left(frac5{32}x^4-frac6{64}x^5right)left(1-x+frac1{64}x^6right)-left(frac1{32}x^5-frac1{64}x^6right)left(-1+frac6{64}x^5right)}{left(1-x+frac1{64}x^6right)^2}right|_{large x=1}\
&=frac{frac4{64}frac1{64}+frac1{64}frac{58}{64}}{left(frac1{64}right)^2}\[12pt]
&=62
end{align}
$$






share|cite|improve this answer









$endgroup$









  • 1




    $begingroup$
    Could you elaborate briefly on why the derivative gives the expected number of flips?
    $endgroup$
    – Austin Mohr
    Aug 20 '13 at 2:37






  • 2




    $begingroup$
    @AustinMohr: If $f(x)$ is the generating function of the probability $p_n$ of the ending after $n$ tosses $$ f(x)=sum_{n=0}^infty p_nx^n $$ then, because the probability of lasting an infinite number of tosses is $0$, we have $$ begin{align} f(1) &=sum_{n=0}^infty p_n\ &=1 end{align} $$ Furthermore, $$ begin{align} f'(1) &=sum_{n=0}^infty n,p_n\ &=mathrm{E}(n) end{align} $$
    $endgroup$
    – robjohn
    Aug 20 '13 at 5:13












  • $begingroup$
    Do you know how to find the distribution (or expectation and variance) for the number of tosses until either 5 consecutive heads or 5 consecutive tails? (Or 5 consecutive equal results from rolling dice.) Is there a question on math.se about this?
    $endgroup$
    – ShreevatsaR
    Dec 15 '15 at 14:17












  • $begingroup$
    I found an answer using martingales here: quora.com/… but I'm curious if there is a generating functions way (also about the distribution, say variance or number of trials until 90% probability of seeing what we want).
    $endgroup$
    – ShreevatsaR
    Dec 15 '15 at 14:31












  • $begingroup$
    @ShreevatsaR: The generating function for the probability of ending on $n$ tosses for that problem is similar: $$frac{frac1{16}x^5}{1-left(frac12x+frac14x^2 +frac18x^3+frac1{16}x^4right)}$$ From that, we can compute the expectation and variance. Looking at the coefficients of the series for the generating function we can also find out that in $66 $ tosses, we will have a $90.0761%$ chance of seeing $5$ heads or $5$ tails in a row.
    $endgroup$
    – robjohn
    Dec 15 '15 at 17:32





















14












$begingroup$

Lets calculate it for $n$ consecutive tosses the expected number of tosses needed.



Lets denote $E_n$ for $n$ consecutive heads.
Now if we get one more head after $E_{n-1}$, then we have $n$ consecutive heads
or if it is a tail then again we have to repeat the procedure.



So for the two scenarios:




  1. $E_{n-1}+1$

  2. $E_{n}{+1}$ ($1$ for a tail)


So, $E_n=frac12(E_{n-1} +1)+frac12(E_{n-1}+ E_n+ 1)$,
so $E_n= 2E_{n-1}+2$.



We have the general recurrence relation. Define $f(n)=E_n+2$ with $f(0)=2$. So,



begin{align}
f(n)&=2f(n-1) \
implies f(n)&=2^{n+1}
end{align}



Therefore, $E_n = 2^{n+1}-2 = 2(2^n-1)$



For $n=5$, it will give us $2(2^5-1)=62$.






share|cite|improve this answer











$endgroup$









  • 1




    $begingroup$
    Amazing solution, thank you
    $endgroup$
    – Jaydev
    Jul 24 '17 at 1:30



















9












$begingroup$

This problem is solvable with the next step conditioning method. Let $mu_k$ denote the mean number of tosses until 5 consecutive heads occurs, given that $k$ consecutive heads just occured. Obviously $mu_5=0$. Conditioning on the outcome of the next coin throw:
$$
mu_k = 1 + frac{1}{2} mu_{k+1} + frac{1}{2} mu_0
$$
Solving the resulting linear system:



In[28]:= Solve[Table[mu[k] == 1 + 1/2 mu[k + 1] + mu[0]/2, {k, 0, 4}],
Table[mu[k], {k, 0, 4}]] /. mu[5] -> 0

Out[28]= {{mu[0] -> 62, mu[1] -> 60, mu[2] -> 56, mu[3] -> 48,
mu[4] -> 32}}


Hence the expected number of coin flips $mu_0$ equals 62.






share|cite|improve this answer









$endgroup$













  • $begingroup$
    What tool did you use for solving?
    $endgroup$
    – pushpen.paul
    May 11 '15 at 17:19










  • $begingroup$
    @pushpen.paul I used Mathematica
    $endgroup$
    – Sasha
    May 11 '15 at 17:20










  • $begingroup$
    Can you please explain the original equation?
    $endgroup$
    – BOS
    Sep 20 '16 at 15:58










  • $begingroup$
    @BOS Since $mu_k$ is the conditional expectation, consider the next coin toss. Because a new toss was made, we add 1, in the next state, with equal probabilities we either get next head, in which case we gonna get $k+1$ heads, hence $mu_{k+1}$, or the tail, in which case we break the streak of consecutive heads, hence $mu_0$.
    $endgroup$
    – Sasha
    Sep 23 '16 at 3:03






  • 1




    $begingroup$
    @Sasha This is the Markov way of solving it, right? This seems most intuitive to me of all methods presented here.
    $endgroup$
    – user3496060
    Dec 22 '17 at 8:06





















2












$begingroup$

The question can be generalized to what is the expected number of tosses before we get x heads.Let's call this E(x). We can easily derive a recursive formula for E(x).
Now, there are a total of two possibilities, first is that we fail to get the xth consecutive heads in xth attempt and second, we succeed. Probability of success is 1/(2^x) and probability of failure is 1-(1/(2^x)).



Now, if we were to fail to get xth consecutive heads in xth toss (i.e. case 1), the we will have to use a total of (E(x)+1) moves, because one move has been wasted.



On the other hand if we were to succeed in getting xth consecutive head in xth toss (i.e. case 2), the total moves is E(x-1)+1 , because we now take one move more than that was required to get x-1 consecutive heads.



So,



E(x) = P(failure) * (E(x)+1) + P(success) * (E(x-1)+1)

E(x) = [1-(1/(2^x))] * (E(x)+1) + [1/(2^x)] * (E(x-1)+1)



Also E(0) = 0 , because expected number of tosses to get 0 heads is zero, duh



now,



E(1) = (1-0.5) * (E(1)+1) + (0.5) * (E(0)+1) =>
E(1) = 2



E(2) = (1-0.125) * (E(1)+1) + (0.125) * (E(1)+1) =>
E(2) = 6



Similarly,



E(3) = 14



E(4) = 30



E(5) = 62






share|cite|improve this answer











$endgroup$





















    1












    $begingroup$

    I would simplify the problem as follows:



    Let $e$ = Expected number of flips until $5$ consecutive $H$, i.e., $E[5H]$

    Let $f$ = Expected number of flips until $5$ consecutive $H$ when we have seen one $H$, i.e., $E[5H|H]$

    Let $g$ = Expected number of flips until $5$ consecutive $H$ when we have seen two $H$, i.e., $E[5H|2H]$

    Let $h$ = Expected number of flips until $5$ consecutive $H$ when we have seen three $H$, i.e., $E[5H|3H]$

    Let $i$ = Expected number of flips until $5$ consecutive $H$ when we have seen four $H$, i.e., $E[5H|4H]$



    Now Start flipping coin, there is $frac{1}{2}$ probability of getting $H$ or $T$. So if we get $H$ then expected number of flips until 5 consecutive $H$ is $(f+1)$. Alternatively if $T$, we wasted 1 flip and expected number is still $(e+1)$
    $$
    e=frac12(e+1)+frac12(f+1);
    $$



    We now need $f$ to solve above to get $e$. Now we start with 1 $H$ and seeking 4 more $H$ to get total 5 $H$. Again, there is $frac{1}{2}$ probability of getting $H$ or $T$. So if we get $H$ (total $2H$ so far) then expected number of flips until 5 consecutive $H$ is $(g+1)$. Alternatively if $T$, we wasted this flip and expected number is back to $(e+1)$



    $$
    f=frac12(g+1)+frac12(e+1);
    $$



    Continuing this way...
    $$
    g=frac12(h+1)+frac12(e+1);
    $$
    $$
    h=frac12(i+1)+frac12(e+1);
    $$



    Finally, Now we have 4 $H$ and seeking last $H$ to get total 5 $H$. Still, there is $frac{1}{2}$ probability of getting $H$ or $T$. So if we get $H$ (total $5H$) then we need just $1$ flip. Alternatively if $T$ is observed, we wasted this flip and expected number is back to $(e+1)$
    $$
    i=frac12(1)+frac12(e+1);
    $$



    Solving these equations, $e=62$, $f=60$, $g=56$, $h=48$, $i=32$

    This solution offers some insight into conditional expectations of number of flips needed till 5 consecutive $H$ given 1, 2, 3 and 4 consecutive $H$.






    share|cite|improve this answer











    $endgroup$





















      0












      $begingroup$

      Expectation for getting n consecutive heads is : 2*(2^n-1). Thus for 5 heads it is = 62.






      share|cite|improve this answer









      $endgroup$









      • 6




        $begingroup$
        How did you get the formula?
        $endgroup$
        – pushpen.paul
        May 11 '15 at 18:25



















      0












      $begingroup$

      No one else seems to have suggested the following approach. Suppose we keep flipping a coin until we get five heads in a row. Define a "run" as either five consecutive heads or a tails flip plus the preceding streak of heads flips. (A run could be a single tails flip.) The number of coin flips is equal to the number of runs with at least four heads ($R_{4+}$), plus the number of runs with at least three heads ($R_{3+}$), and so on down to the number of runs with at least zero heads ($R_{0+}$). We can see this by expanding the terms:



      $R_{0+}$ = # runs with 0 heads + # runs with 1 head + ... + # runs with 5 heads



      $R_{1+}$ = # runs with 1 head + # runs with 2 heads + ... + # runs with 5 heads



      ...



      $R_{4+}$ = # runs with 4 heads + # runs with 5 heads



      # flips = # flips in runs with 0 heads + # flips in runs with 1 head + ... + # flips in runs with 5 heads



      # flips in runs with 0 heads = # runs with 0 heads



      # flips in runs with 1 head = 2 x # runs with 1 head



      ...



      # flips in runs with 4 heads = 5 x # runs with 4 heads



      # flips in runs with 5 heads = 5 x # runs with 5 heads



      By linearity of expectation, the expected number of coin flips is $E(R_{0+}) + E(R_{1+}) + ldots + E(R_{4+})$. $E(R_{4+})$ is $2 E(R_{5+}) = 2$, because one half of the time we flip at least four heads in a row, we go on to flip five heads in a row, i.e. the following coin flip is heads. In other words, in expectation, it takes two runs that start with four heads to achieve one run of five heads. Likewise, $E(R_{3+}) = 2 E(R_{4+}) = 4$, $E(R_{2+}) = 2 E(R_{3+}) = 8$, $E(R_{1+}) = 2 E(R_{2+}) = 16$, and $E(R_{0+}) = 2 E(R_{1+}) = 32$. The expected number of coin flips is $32 + 16 + 8 + 4 + 2 = 62$.



      More generally, given a biased coin that comes up heads $p$ portion of the time, the expected number of flips to get $n$ heads in a row is $frac{1}{p} + frac{1}{p^2} + ldots + frac{1}{p^n} = frac{1 - p^n}{p^n(1 - p)}$.






      share|cite|improve this answer









      $endgroup$





















        0












        $begingroup$

        We can solve this without equations. Ask the following (auxiliary) question: how many flips you need to get either $N$ heads or $N$ tails. Then to get $N$ heads only you need twice as many flips. Start with the question of how many flips you need to get either $H$ or $T$. The answer is
        $$x = frac12 (1) + frac12 (1) = 1.$$
        The reason is that there is $frac12$ probability to get $H$, and after $1$ flip you are done. The same for $T$. To get only one $H$ you then need two flips. OK. Now we ask what it takes to get $HH$ or $TT$. The result is
        $$x=frac12(1+2) + frac12(1+2).$$ The number $2$ appears because, say you flip $H$ first, then you need on average $2$ flips to get another $H$, as we learned earlier. The same for $T$. So you need $3$ flips to get $TT$ or $HH$, and you need $6$ flips to get $HH$ only. And so on. You need $frac12 (1+6) + frac12(1+6) = 7$ flips to get either $HHH$ or $TTT$, and $14$ to get $HHH$ only. If you need $HHHH$ or $TTTT$, then flip $frac12(1+14) + frac12(1+14) = 15$ times, or $30$ times to get just $HHHH$. The sequence is $1, 3, 7, 15, ldots$ to get either heads or tails. The formula is easy to extract: you need $2^N-1$ flips to get either $N$ heads or $N$ tails, or $2^{N+1}-2$ to get $N$ heads only. If $N=5$ we get the answer: $62$.






        share|cite|improve this answer











        $endgroup$





















          0












          $begingroup$

          Use Markov chains. The nice part of Markov Chains is that they can be applied to a huge class of similar problems with relatively little thought (it's almost formulaic in application). It's also the most intuitive way to handle these problems.



          (in Matlab code notation below)



          %% setup full transition matrix with states from zero heads to 5 heads



          T = $[ones(5,1)*.5,eye(5)*.5];$



          $T = [T;zeros(1,6)]$



          %%Take subset "Q" comprised of just transient states (5 heads is absorbing state)
          $Q = T(1:end-1,1:end-1);$



          $M = inv(eye(5)-Q)$



          absorbing Markov Chain has a similar example as this question BTW...



          ans =



          62
          60
          56
          48
          32


          Where each row is the expected number of steps before being absorbed when starting in that transient state (0 through 4 heads, top to bottom).






          share|cite|improve this answer









          $endgroup$













            Your Answer





            StackExchange.ifUsing("editor", function () {
            return StackExchange.using("mathjaxEditing", function () {
            StackExchange.MarkdownEditor.creationCallbacks.add(function (editor, postfix) {
            StackExchange.mathjaxEditing.prepareWmdForMathJax(editor, postfix, [["$", "$"], ["\\(","\\)"]]);
            });
            });
            }, "mathjax-editing");

            StackExchange.ready(function() {
            var channelOptions = {
            tags: "".split(" "),
            id: "69"
            };
            initTagRenderer("".split(" "), "".split(" "), channelOptions);

            StackExchange.using("externalEditor", function() {
            // Have to fire editor after snippets, if snippets enabled
            if (StackExchange.settings.snippets.snippetsEnabled) {
            StackExchange.using("snippets", function() {
            createEditor();
            });
            }
            else {
            createEditor();
            }
            });

            function createEditor() {
            StackExchange.prepareEditor({
            heartbeatType: 'answer',
            autoActivateHeartbeat: false,
            convertImagesToLinks: true,
            noModals: true,
            showLowRepImageUploadWarning: true,
            reputationToPostImages: 10,
            bindNavPrevention: true,
            postfix: "",
            imageUploader: {
            brandingHtml: "Powered by u003ca class="icon-imgur-white" href="https://imgur.com/"u003eu003c/au003e",
            contentPolicyHtml: "User contributions licensed under u003ca href="https://creativecommons.org/licenses/by-sa/3.0/"u003ecc by-sa 3.0 with attribution requiredu003c/au003e u003ca href="https://stackoverflow.com/legal/content-policy"u003e(content policy)u003c/au003e",
            allowUrls: true
            },
            noCode: true, onDemand: true,
            discardSelector: ".discard-answer"
            ,immediatelyShowMarkdownHelp:true
            });


            }
            });














            draft saved

            draft discarded


















            StackExchange.ready(
            function () {
            StackExchange.openid.initPostLogin('.new-post-login', 'https%3a%2f%2fmath.stackexchange.com%2fquestions%2f364038%2fexpected-number-of-coin-tosses-to-get-five-consecutive-heads%23new-answer', 'question_page');
            }
            );

            Post as a guest















            Required, but never shown

























            10 Answers
            10






            active

            oldest

            votes








            10 Answers
            10






            active

            oldest

            votes









            active

            oldest

            votes






            active

            oldest

            votes









            77












            $begingroup$

            Let $e$ be the expected number of tosses. It is clear that $e$ is finite.



            Start tossing. If we get a tail immediately (probability $frac{1}{2}$) then the expected number is $e+1$. If we get a head then a tail (probability $frac{1}{4}$), then the expected number is $e+2$. Continue $dots$. If we get $4$ heads then a tail, the expected number is $e+5$. Finally, if our first $5$ tosses are heads, then the expected number is $5$. Thus
            $$e=frac{1}{2}(e+1)+frac{1}{4}(e+2)+frac{1}{8}(e+3)+frac{1}{16}(e+4)+frac{1}{32}(e+5)+frac{1}{32}(5).$$
            Solve this linear equation for $e$. We get $e=62$.






            share|cite|improve this answer









            $endgroup$









            • 1




              $begingroup$
              It is clear that e is finite, but how can you show it properly though ? Thanks.
              $endgroup$
              – Dark
              Jul 3 '15 at 17:39






            • 2




              $begingroup$
              If one wants, let $X$ be the number of tosses. Then $Pr(X=n)le (1/2)^{n-5}$. So $E(X)le sum n (1/2)^{n-5}$, a convergent series.
              $endgroup$
              – André Nicolas
              Jul 3 '15 at 17:58








            • 14




              $begingroup$
              The same method obviously generalizes to give $e_n$, the expected number of tosses to get $n$ consecutive heads ($n ge 1$): $$e_n=frac{1}{2}(e_n+1)+frac{1}{4}(e_n+2)+frac{1}{8}(e_n+3)+frac{1}{16}(e_n+4)+cdots +frac{1}{2^n}(e_n+n)+frac{1}{2^n}(n),$$ the solution of which is easily found to be $$e_n = 2(2^n - 1).$$
              $endgroup$
              – r.e.s.
              Jul 19 '15 at 23:57






            • 4




              $begingroup$
              Why are TT, TTT not considered?
              $endgroup$
              – Jaydev
              Jul 24 '17 at 1:24






            • 4




              $begingroup$
              @Jaydev TT and TTT both are covered by the case "if we get a tail immediately".
              $endgroup$
              – David K
              Oct 6 '17 at 13:13
















            77












            $begingroup$

            Let $e$ be the expected number of tosses. It is clear that $e$ is finite.



            Start tossing. If we get a tail immediately (probability $frac{1}{2}$) then the expected number is $e+1$. If we get a head then a tail (probability $frac{1}{4}$), then the expected number is $e+2$. Continue $dots$. If we get $4$ heads then a tail, the expected number is $e+5$. Finally, if our first $5$ tosses are heads, then the expected number is $5$. Thus
            $$e=frac{1}{2}(e+1)+frac{1}{4}(e+2)+frac{1}{8}(e+3)+frac{1}{16}(e+4)+frac{1}{32}(e+5)+frac{1}{32}(5).$$
            Solve this linear equation for $e$. We get $e=62$.






            share|cite|improve this answer









            $endgroup$









            • 1




              $begingroup$
              It is clear that e is finite, but how can you show it properly though ? Thanks.
              $endgroup$
              – Dark
              Jul 3 '15 at 17:39






            • 2




              $begingroup$
              If one wants, let $X$ be the number of tosses. Then $Pr(X=n)le (1/2)^{n-5}$. So $E(X)le sum n (1/2)^{n-5}$, a convergent series.
              $endgroup$
              – André Nicolas
              Jul 3 '15 at 17:58








            • 14




              $begingroup$
              The same method obviously generalizes to give $e_n$, the expected number of tosses to get $n$ consecutive heads ($n ge 1$): $$e_n=frac{1}{2}(e_n+1)+frac{1}{4}(e_n+2)+frac{1}{8}(e_n+3)+frac{1}{16}(e_n+4)+cdots +frac{1}{2^n}(e_n+n)+frac{1}{2^n}(n),$$ the solution of which is easily found to be $$e_n = 2(2^n - 1).$$
              $endgroup$
              – r.e.s.
              Jul 19 '15 at 23:57






            • 4




              $begingroup$
              Why are TT, TTT not considered?
              $endgroup$
              – Jaydev
              Jul 24 '17 at 1:24






            • 4




              $begingroup$
              @Jaydev TT and TTT both are covered by the case "if we get a tail immediately".
              $endgroup$
              – David K
              Oct 6 '17 at 13:13














            77












            77








            77





            $begingroup$

            Let $e$ be the expected number of tosses. It is clear that $e$ is finite.



            Start tossing. If we get a tail immediately (probability $frac{1}{2}$) then the expected number is $e+1$. If we get a head then a tail (probability $frac{1}{4}$), then the expected number is $e+2$. Continue $dots$. If we get $4$ heads then a tail, the expected number is $e+5$. Finally, if our first $5$ tosses are heads, then the expected number is $5$. Thus
            $$e=frac{1}{2}(e+1)+frac{1}{4}(e+2)+frac{1}{8}(e+3)+frac{1}{16}(e+4)+frac{1}{32}(e+5)+frac{1}{32}(5).$$
            Solve this linear equation for $e$. We get $e=62$.






            share|cite|improve this answer









            $endgroup$



            Let $e$ be the expected number of tosses. It is clear that $e$ is finite.



            Start tossing. If we get a tail immediately (probability $frac{1}{2}$) then the expected number is $e+1$. If we get a head then a tail (probability $frac{1}{4}$), then the expected number is $e+2$. Continue $dots$. If we get $4$ heads then a tail, the expected number is $e+5$. Finally, if our first $5$ tosses are heads, then the expected number is $5$. Thus
            $$e=frac{1}{2}(e+1)+frac{1}{4}(e+2)+frac{1}{8}(e+3)+frac{1}{16}(e+4)+frac{1}{32}(e+5)+frac{1}{32}(5).$$
            Solve this linear equation for $e$. We get $e=62$.







            share|cite|improve this answer












            share|cite|improve this answer



            share|cite|improve this answer










            answered Apr 17 '13 at 5:46









            André NicolasAndré Nicolas

            454k36432819




            454k36432819








            • 1




              $begingroup$
              It is clear that e is finite, but how can you show it properly though ? Thanks.
              $endgroup$
              – Dark
              Jul 3 '15 at 17:39






            • 2




              $begingroup$
              If one wants, let $X$ be the number of tosses. Then $Pr(X=n)le (1/2)^{n-5}$. So $E(X)le sum n (1/2)^{n-5}$, a convergent series.
              $endgroup$
              – André Nicolas
              Jul 3 '15 at 17:58








            • 14




              $begingroup$
              The same method obviously generalizes to give $e_n$, the expected number of tosses to get $n$ consecutive heads ($n ge 1$): $$e_n=frac{1}{2}(e_n+1)+frac{1}{4}(e_n+2)+frac{1}{8}(e_n+3)+frac{1}{16}(e_n+4)+cdots +frac{1}{2^n}(e_n+n)+frac{1}{2^n}(n),$$ the solution of which is easily found to be $$e_n = 2(2^n - 1).$$
              $endgroup$
              – r.e.s.
              Jul 19 '15 at 23:57






            • 4




              $begingroup$
              Why are TT, TTT not considered?
              $endgroup$
              – Jaydev
              Jul 24 '17 at 1:24






            • 4




              $begingroup$
              @Jaydev TT and TTT both are covered by the case "if we get a tail immediately".
              $endgroup$
              – David K
              Oct 6 '17 at 13:13














            • 1




              $begingroup$
              It is clear that e is finite, but how can you show it properly though ? Thanks.
              $endgroup$
              – Dark
              Jul 3 '15 at 17:39






            • 2




              $begingroup$
              If one wants, let $X$ be the number of tosses. Then $Pr(X=n)le (1/2)^{n-5}$. So $E(X)le sum n (1/2)^{n-5}$, a convergent series.
              $endgroup$
              – André Nicolas
              Jul 3 '15 at 17:58








            • 14




              $begingroup$
              The same method obviously generalizes to give $e_n$, the expected number of tosses to get $n$ consecutive heads ($n ge 1$): $$e_n=frac{1}{2}(e_n+1)+frac{1}{4}(e_n+2)+frac{1}{8}(e_n+3)+frac{1}{16}(e_n+4)+cdots +frac{1}{2^n}(e_n+n)+frac{1}{2^n}(n),$$ the solution of which is easily found to be $$e_n = 2(2^n - 1).$$
              $endgroup$
              – r.e.s.
              Jul 19 '15 at 23:57






            • 4




              $begingroup$
              Why are TT, TTT not considered?
              $endgroup$
              – Jaydev
              Jul 24 '17 at 1:24






            • 4




              $begingroup$
              @Jaydev TT and TTT both are covered by the case "if we get a tail immediately".
              $endgroup$
              – David K
              Oct 6 '17 at 13:13








            1




            1




            $begingroup$
            It is clear that e is finite, but how can you show it properly though ? Thanks.
            $endgroup$
            – Dark
            Jul 3 '15 at 17:39




            $begingroup$
            It is clear that e is finite, but how can you show it properly though ? Thanks.
            $endgroup$
            – Dark
            Jul 3 '15 at 17:39




            2




            2




            $begingroup$
            If one wants, let $X$ be the number of tosses. Then $Pr(X=n)le (1/2)^{n-5}$. So $E(X)le sum n (1/2)^{n-5}$, a convergent series.
            $endgroup$
            – André Nicolas
            Jul 3 '15 at 17:58






            $begingroup$
            If one wants, let $X$ be the number of tosses. Then $Pr(X=n)le (1/2)^{n-5}$. So $E(X)le sum n (1/2)^{n-5}$, a convergent series.
            $endgroup$
            – André Nicolas
            Jul 3 '15 at 17:58






            14




            14




            $begingroup$
            The same method obviously generalizes to give $e_n$, the expected number of tosses to get $n$ consecutive heads ($n ge 1$): $$e_n=frac{1}{2}(e_n+1)+frac{1}{4}(e_n+2)+frac{1}{8}(e_n+3)+frac{1}{16}(e_n+4)+cdots +frac{1}{2^n}(e_n+n)+frac{1}{2^n}(n),$$ the solution of which is easily found to be $$e_n = 2(2^n - 1).$$
            $endgroup$
            – r.e.s.
            Jul 19 '15 at 23:57




            $begingroup$
            The same method obviously generalizes to give $e_n$, the expected number of tosses to get $n$ consecutive heads ($n ge 1$): $$e_n=frac{1}{2}(e_n+1)+frac{1}{4}(e_n+2)+frac{1}{8}(e_n+3)+frac{1}{16}(e_n+4)+cdots +frac{1}{2^n}(e_n+n)+frac{1}{2^n}(n),$$ the solution of which is easily found to be $$e_n = 2(2^n - 1).$$
            $endgroup$
            – r.e.s.
            Jul 19 '15 at 23:57




            4




            4




            $begingroup$
            Why are TT, TTT not considered?
            $endgroup$
            – Jaydev
            Jul 24 '17 at 1:24




            $begingroup$
            Why are TT, TTT not considered?
            $endgroup$
            – Jaydev
            Jul 24 '17 at 1:24




            4




            4




            $begingroup$
            @Jaydev TT and TTT both are covered by the case "if we get a tail immediately".
            $endgroup$
            – David K
            Oct 6 '17 at 13:13




            $begingroup$
            @Jaydev TT and TTT both are covered by the case "if we get a tail immediately".
            $endgroup$
            – David K
            Oct 6 '17 at 13:13











            21












            $begingroup$

            Here is a generating function approach.



            Consider the following toss strings, probabilities, and terms



            $$
            color{#00A000}{
            begin{array}{llc}
            T&frac12&qquadfrac12x\
            HT&frac14&qquadfrac14x^2\
            HHT&frac18&qquadfrac18x^3\
            HHHT&frac1{16}&qquadfrac1{16}x^4\
            HHHHT&frac1{32}&qquadfrac1{32}x^5\
            color{#C00000}{HHHHH}&color{#C00000}{frac1{32}}&color{#C00000}{qquadfrac1{32}x^5}
            end{array}
            }
            $$
            Each term has the probability as its coefficient and the length of the string as its exponent.



            Possible outcomes are any combination of the green strings followed by the red string. We get the generating function of the probability of ending after $n$ tosses to be
            $$
            begin{align}
            f(x)&=sum_{k=0}^inftyleft(frac12x+frac14x^2+frac18x^3+frac1{16}x^4+frac1{32}x^5right)^kfrac1{32}x^5\
            &=frac{frac1{32}x^5}{1-left(frac12x+frac14x^2+frac18x^3+frac1{16}x^4+frac1{32}x^5right)}\
            &=frac{frac1{32}x^5}{1-frac{frac12x-frac1{64}x^6}{1-frac12x}}\
            &=frac{frac1{32}x^5-frac1{64}x^6}{1-x+frac1{64}x^6}
            end{align}
            $$
            The average duration is then
            $$
            begin{align}
            f'(1)
            &=left.frac{left(frac5{32}x^4-frac6{64}x^5right)left(1-x+frac1{64}x^6right)-left(frac1{32}x^5-frac1{64}x^6right)left(-1+frac6{64}x^5right)}{left(1-x+frac1{64}x^6right)^2}right|_{large x=1}\
            &=frac{frac4{64}frac1{64}+frac1{64}frac{58}{64}}{left(frac1{64}right)^2}\[12pt]
            &=62
            end{align}
            $$






            share|cite|improve this answer









            $endgroup$









            • 1




              $begingroup$
              Could you elaborate briefly on why the derivative gives the expected number of flips?
              $endgroup$
              – Austin Mohr
              Aug 20 '13 at 2:37






            • 2




              $begingroup$
              @AustinMohr: If $f(x)$ is the generating function of the probability $p_n$ of the ending after $n$ tosses $$ f(x)=sum_{n=0}^infty p_nx^n $$ then, because the probability of lasting an infinite number of tosses is $0$, we have $$ begin{align} f(1) &=sum_{n=0}^infty p_n\ &=1 end{align} $$ Furthermore, $$ begin{align} f'(1) &=sum_{n=0}^infty n,p_n\ &=mathrm{E}(n) end{align} $$
              $endgroup$
              – robjohn
              Aug 20 '13 at 5:13












            • $begingroup$
              Do you know how to find the distribution (or expectation and variance) for the number of tosses until either 5 consecutive heads or 5 consecutive tails? (Or 5 consecutive equal results from rolling dice.) Is there a question on math.se about this?
              $endgroup$
              – ShreevatsaR
              Dec 15 '15 at 14:17












            • $begingroup$
              I found an answer using martingales here: quora.com/… but I'm curious if there is a generating functions way (also about the distribution, say variance or number of trials until 90% probability of seeing what we want).
              $endgroup$
              – ShreevatsaR
              Dec 15 '15 at 14:31












            • $begingroup$
              @ShreevatsaR: The generating function for the probability of ending on $n$ tosses for that problem is similar: $$frac{frac1{16}x^5}{1-left(frac12x+frac14x^2 +frac18x^3+frac1{16}x^4right)}$$ From that, we can compute the expectation and variance. Looking at the coefficients of the series for the generating function we can also find out that in $66 $ tosses, we will have a $90.0761%$ chance of seeing $5$ heads or $5$ tails in a row.
              $endgroup$
              – robjohn
              Dec 15 '15 at 17:32


















            21












            $begingroup$

            Here is a generating function approach.



            Consider the following toss strings, probabilities, and terms



            $$
            color{#00A000}{
            begin{array}{llc}
            T&frac12&qquadfrac12x\
            HT&frac14&qquadfrac14x^2\
            HHT&frac18&qquadfrac18x^3\
            HHHT&frac1{16}&qquadfrac1{16}x^4\
            HHHHT&frac1{32}&qquadfrac1{32}x^5\
            color{#C00000}{HHHHH}&color{#C00000}{frac1{32}}&color{#C00000}{qquadfrac1{32}x^5}
            end{array}
            }
            $$
            Each term has the probability as its coefficient and the length of the string as its exponent.



            Possible outcomes are any combination of the green strings followed by the red string. We get the generating function of the probability of ending after $n$ tosses to be
            $$
            begin{align}
            f(x)&=sum_{k=0}^inftyleft(frac12x+frac14x^2+frac18x^3+frac1{16}x^4+frac1{32}x^5right)^kfrac1{32}x^5\
            &=frac{frac1{32}x^5}{1-left(frac12x+frac14x^2+frac18x^3+frac1{16}x^4+frac1{32}x^5right)}\
            &=frac{frac1{32}x^5}{1-frac{frac12x-frac1{64}x^6}{1-frac12x}}\
            &=frac{frac1{32}x^5-frac1{64}x^6}{1-x+frac1{64}x^6}
            end{align}
            $$
            The average duration is then
            $$
            begin{align}
            f'(1)
            &=left.frac{left(frac5{32}x^4-frac6{64}x^5right)left(1-x+frac1{64}x^6right)-left(frac1{32}x^5-frac1{64}x^6right)left(-1+frac6{64}x^5right)}{left(1-x+frac1{64}x^6right)^2}right|_{large x=1}\
            &=frac{frac4{64}frac1{64}+frac1{64}frac{58}{64}}{left(frac1{64}right)^2}\[12pt]
            &=62
            end{align}
            $$






            share|cite|improve this answer









            $endgroup$









            • 1




              $begingroup$
              Could you elaborate briefly on why the derivative gives the expected number of flips?
              $endgroup$
              – Austin Mohr
              Aug 20 '13 at 2:37






            • 2




              $begingroup$
              @AustinMohr: If $f(x)$ is the generating function of the probability $p_n$ of the ending after $n$ tosses $$ f(x)=sum_{n=0}^infty p_nx^n $$ then, because the probability of lasting an infinite number of tosses is $0$, we have $$ begin{align} f(1) &=sum_{n=0}^infty p_n\ &=1 end{align} $$ Furthermore, $$ begin{align} f'(1) &=sum_{n=0}^infty n,p_n\ &=mathrm{E}(n) end{align} $$
              $endgroup$
              – robjohn
              Aug 20 '13 at 5:13












            • $begingroup$
              Do you know how to find the distribution (or expectation and variance) for the number of tosses until either 5 consecutive heads or 5 consecutive tails? (Or 5 consecutive equal results from rolling dice.) Is there a question on math.se about this?
              $endgroup$
              – ShreevatsaR
              Dec 15 '15 at 14:17












            • $begingroup$
              I found an answer using martingales here: quora.com/… but I'm curious if there is a generating functions way (also about the distribution, say variance or number of trials until 90% probability of seeing what we want).
              $endgroup$
              – ShreevatsaR
              Dec 15 '15 at 14:31












            • $begingroup$
              @ShreevatsaR: The generating function for the probability of ending on $n$ tosses for that problem is similar: $$frac{frac1{16}x^5}{1-left(frac12x+frac14x^2 +frac18x^3+frac1{16}x^4right)}$$ From that, we can compute the expectation and variance. Looking at the coefficients of the series for the generating function we can also find out that in $66 $ tosses, we will have a $90.0761%$ chance of seeing $5$ heads or $5$ tails in a row.
              $endgroup$
              – robjohn
              Dec 15 '15 at 17:32
















            21












            21








            21





            $begingroup$

            Here is a generating function approach.



            Consider the following toss strings, probabilities, and terms



            $$
            color{#00A000}{
            begin{array}{llc}
            T&frac12&qquadfrac12x\
            HT&frac14&qquadfrac14x^2\
            HHT&frac18&qquadfrac18x^3\
            HHHT&frac1{16}&qquadfrac1{16}x^4\
            HHHHT&frac1{32}&qquadfrac1{32}x^5\
            color{#C00000}{HHHHH}&color{#C00000}{frac1{32}}&color{#C00000}{qquadfrac1{32}x^5}
            end{array}
            }
            $$
            Each term has the probability as its coefficient and the length of the string as its exponent.



            Possible outcomes are any combination of the green strings followed by the red string. We get the generating function of the probability of ending after $n$ tosses to be
            $$
            begin{align}
            f(x)&=sum_{k=0}^inftyleft(frac12x+frac14x^2+frac18x^3+frac1{16}x^4+frac1{32}x^5right)^kfrac1{32}x^5\
            &=frac{frac1{32}x^5}{1-left(frac12x+frac14x^2+frac18x^3+frac1{16}x^4+frac1{32}x^5right)}\
            &=frac{frac1{32}x^5}{1-frac{frac12x-frac1{64}x^6}{1-frac12x}}\
            &=frac{frac1{32}x^5-frac1{64}x^6}{1-x+frac1{64}x^6}
            end{align}
            $$
            The average duration is then
            $$
            begin{align}
            f'(1)
            &=left.frac{left(frac5{32}x^4-frac6{64}x^5right)left(1-x+frac1{64}x^6right)-left(frac1{32}x^5-frac1{64}x^6right)left(-1+frac6{64}x^5right)}{left(1-x+frac1{64}x^6right)^2}right|_{large x=1}\
            &=frac{frac4{64}frac1{64}+frac1{64}frac{58}{64}}{left(frac1{64}right)^2}\[12pt]
            &=62
            end{align}
            $$






            share|cite|improve this answer









            $endgroup$



            Here is a generating function approach.



            Consider the following toss strings, probabilities, and terms



            $$
            color{#00A000}{
            begin{array}{llc}
            T&frac12&qquadfrac12x\
            HT&frac14&qquadfrac14x^2\
            HHT&frac18&qquadfrac18x^3\
            HHHT&frac1{16}&qquadfrac1{16}x^4\
            HHHHT&frac1{32}&qquadfrac1{32}x^5\
            color{#C00000}{HHHHH}&color{#C00000}{frac1{32}}&color{#C00000}{qquadfrac1{32}x^5}
            end{array}
            }
            $$
            Each term has the probability as its coefficient and the length of the string as its exponent.



            Possible outcomes are any combination of the green strings followed by the red string. We get the generating function of the probability of ending after $n$ tosses to be
            $$
            begin{align}
            f(x)&=sum_{k=0}^inftyleft(frac12x+frac14x^2+frac18x^3+frac1{16}x^4+frac1{32}x^5right)^kfrac1{32}x^5\
            &=frac{frac1{32}x^5}{1-left(frac12x+frac14x^2+frac18x^3+frac1{16}x^4+frac1{32}x^5right)}\
            &=frac{frac1{32}x^5}{1-frac{frac12x-frac1{64}x^6}{1-frac12x}}\
            &=frac{frac1{32}x^5-frac1{64}x^6}{1-x+frac1{64}x^6}
            end{align}
            $$
            The average duration is then
            $$
            begin{align}
            f'(1)
            &=left.frac{left(frac5{32}x^4-frac6{64}x^5right)left(1-x+frac1{64}x^6right)-left(frac1{32}x^5-frac1{64}x^6right)left(-1+frac6{64}x^5right)}{left(1-x+frac1{64}x^6right)^2}right|_{large x=1}\
            &=frac{frac4{64}frac1{64}+frac1{64}frac{58}{64}}{left(frac1{64}right)^2}\[12pt]
            &=62
            end{align}
            $$







            share|cite|improve this answer












            share|cite|improve this answer



            share|cite|improve this answer










            answered Apr 17 '13 at 8:43









            robjohnrobjohn

            270k27312639




            270k27312639








            • 1




              $begingroup$
              Could you elaborate briefly on why the derivative gives the expected number of flips?
              $endgroup$
              – Austin Mohr
              Aug 20 '13 at 2:37






            • 2




              $begingroup$
              @AustinMohr: If $f(x)$ is the generating function of the probability $p_n$ of the ending after $n$ tosses $$ f(x)=sum_{n=0}^infty p_nx^n $$ then, because the probability of lasting an infinite number of tosses is $0$, we have $$ begin{align} f(1) &=sum_{n=0}^infty p_n\ &=1 end{align} $$ Furthermore, $$ begin{align} f'(1) &=sum_{n=0}^infty n,p_n\ &=mathrm{E}(n) end{align} $$
              $endgroup$
              – robjohn
              Aug 20 '13 at 5:13












            • $begingroup$
              Do you know how to find the distribution (or expectation and variance) for the number of tosses until either 5 consecutive heads or 5 consecutive tails? (Or 5 consecutive equal results from rolling dice.) Is there a question on math.se about this?
              $endgroup$
              – ShreevatsaR
              Dec 15 '15 at 14:17












            • $begingroup$
              I found an answer using martingales here: quora.com/… but I'm curious if there is a generating functions way (also about the distribution, say variance or number of trials until 90% probability of seeing what we want).
              $endgroup$
              – ShreevatsaR
              Dec 15 '15 at 14:31












            • $begingroup$
              @ShreevatsaR: The generating function for the probability of ending on $n$ tosses for that problem is similar: $$frac{frac1{16}x^5}{1-left(frac12x+frac14x^2 +frac18x^3+frac1{16}x^4right)}$$ From that, we can compute the expectation and variance. Looking at the coefficients of the series for the generating function we can also find out that in $66 $ tosses, we will have a $90.0761%$ chance of seeing $5$ heads or $5$ tails in a row.
              $endgroup$
              – robjohn
              Dec 15 '15 at 17:32
















            • 1




              $begingroup$
              Could you elaborate briefly on why the derivative gives the expected number of flips?
              $endgroup$
              – Austin Mohr
              Aug 20 '13 at 2:37






            • 2




              $begingroup$
              @AustinMohr: If $f(x)$ is the generating function of the probability $p_n$ of the ending after $n$ tosses $$ f(x)=sum_{n=0}^infty p_nx^n $$ then, because the probability of lasting an infinite number of tosses is $0$, we have $$ begin{align} f(1) &=sum_{n=0}^infty p_n\ &=1 end{align} $$ Furthermore, $$ begin{align} f'(1) &=sum_{n=0}^infty n,p_n\ &=mathrm{E}(n) end{align} $$
              $endgroup$
              – robjohn
              Aug 20 '13 at 5:13












            • $begingroup$
              Do you know how to find the distribution (or expectation and variance) for the number of tosses until either 5 consecutive heads or 5 consecutive tails? (Or 5 consecutive equal results from rolling dice.) Is there a question on math.se about this?
              $endgroup$
              – ShreevatsaR
              Dec 15 '15 at 14:17












            • $begingroup$
              I found an answer using martingales here: quora.com/… but I'm curious if there is a generating functions way (also about the distribution, say variance or number of trials until 90% probability of seeing what we want).
              $endgroup$
              – ShreevatsaR
              Dec 15 '15 at 14:31












            • $begingroup$
              @ShreevatsaR: The generating function for the probability of ending on $n$ tosses for that problem is similar: $$frac{frac1{16}x^5}{1-left(frac12x+frac14x^2 +frac18x^3+frac1{16}x^4right)}$$ From that, we can compute the expectation and variance. Looking at the coefficients of the series for the generating function we can also find out that in $66 $ tosses, we will have a $90.0761%$ chance of seeing $5$ heads or $5$ tails in a row.
              $endgroup$
              – robjohn
              Dec 15 '15 at 17:32










            1




            1




            $begingroup$
            Could you elaborate briefly on why the derivative gives the expected number of flips?
            $endgroup$
            – Austin Mohr
            Aug 20 '13 at 2:37




            $begingroup$
            Could you elaborate briefly on why the derivative gives the expected number of flips?
            $endgroup$
            – Austin Mohr
            Aug 20 '13 at 2:37




            2




            2




            $begingroup$
            @AustinMohr: If $f(x)$ is the generating function of the probability $p_n$ of the ending after $n$ tosses $$ f(x)=sum_{n=0}^infty p_nx^n $$ then, because the probability of lasting an infinite number of tosses is $0$, we have $$ begin{align} f(1) &=sum_{n=0}^infty p_n\ &=1 end{align} $$ Furthermore, $$ begin{align} f'(1) &=sum_{n=0}^infty n,p_n\ &=mathrm{E}(n) end{align} $$
            $endgroup$
            – robjohn
            Aug 20 '13 at 5:13






            $begingroup$
            @AustinMohr: If $f(x)$ is the generating function of the probability $p_n$ of the ending after $n$ tosses $$ f(x)=sum_{n=0}^infty p_nx^n $$ then, because the probability of lasting an infinite number of tosses is $0$, we have $$ begin{align} f(1) &=sum_{n=0}^infty p_n\ &=1 end{align} $$ Furthermore, $$ begin{align} f'(1) &=sum_{n=0}^infty n,p_n\ &=mathrm{E}(n) end{align} $$
            $endgroup$
            – robjohn
            Aug 20 '13 at 5:13














            $begingroup$
            Do you know how to find the distribution (or expectation and variance) for the number of tosses until either 5 consecutive heads or 5 consecutive tails? (Or 5 consecutive equal results from rolling dice.) Is there a question on math.se about this?
            $endgroup$
            – ShreevatsaR
            Dec 15 '15 at 14:17






            $begingroup$
            Do you know how to find the distribution (or expectation and variance) for the number of tosses until either 5 consecutive heads or 5 consecutive tails? (Or 5 consecutive equal results from rolling dice.) Is there a question on math.se about this?
            $endgroup$
            – ShreevatsaR
            Dec 15 '15 at 14:17














            $begingroup$
            I found an answer using martingales here: quora.com/… but I'm curious if there is a generating functions way (also about the distribution, say variance or number of trials until 90% probability of seeing what we want).
            $endgroup$
            – ShreevatsaR
            Dec 15 '15 at 14:31






            $begingroup$
            I found an answer using martingales here: quora.com/… but I'm curious if there is a generating functions way (also about the distribution, say variance or number of trials until 90% probability of seeing what we want).
            $endgroup$
            – ShreevatsaR
            Dec 15 '15 at 14:31














            $begingroup$
            @ShreevatsaR: The generating function for the probability of ending on $n$ tosses for that problem is similar: $$frac{frac1{16}x^5}{1-left(frac12x+frac14x^2 +frac18x^3+frac1{16}x^4right)}$$ From that, we can compute the expectation and variance. Looking at the coefficients of the series for the generating function we can also find out that in $66 $ tosses, we will have a $90.0761%$ chance of seeing $5$ heads or $5$ tails in a row.
            $endgroup$
            – robjohn
            Dec 15 '15 at 17:32






            $begingroup$
            @ShreevatsaR: The generating function for the probability of ending on $n$ tosses for that problem is similar: $$frac{frac1{16}x^5}{1-left(frac12x+frac14x^2 +frac18x^3+frac1{16}x^4right)}$$ From that, we can compute the expectation and variance. Looking at the coefficients of the series for the generating function we can also find out that in $66 $ tosses, we will have a $90.0761%$ chance of seeing $5$ heads or $5$ tails in a row.
            $endgroup$
            – robjohn
            Dec 15 '15 at 17:32













            14












            $begingroup$

            Lets calculate it for $n$ consecutive tosses the expected number of tosses needed.



            Lets denote $E_n$ for $n$ consecutive heads.
            Now if we get one more head after $E_{n-1}$, then we have $n$ consecutive heads
            or if it is a tail then again we have to repeat the procedure.



            So for the two scenarios:




            1. $E_{n-1}+1$

            2. $E_{n}{+1}$ ($1$ for a tail)


            So, $E_n=frac12(E_{n-1} +1)+frac12(E_{n-1}+ E_n+ 1)$,
            so $E_n= 2E_{n-1}+2$.



            We have the general recurrence relation. Define $f(n)=E_n+2$ with $f(0)=2$. So,



            begin{align}
            f(n)&=2f(n-1) \
            implies f(n)&=2^{n+1}
            end{align}



            Therefore, $E_n = 2^{n+1}-2 = 2(2^n-1)$



            For $n=5$, it will give us $2(2^5-1)=62$.






            share|cite|improve this answer











            $endgroup$









            • 1




              $begingroup$
              Amazing solution, thank you
              $endgroup$
              – Jaydev
              Jul 24 '17 at 1:30
















            14












            $begingroup$

            Lets calculate it for $n$ consecutive tosses the expected number of tosses needed.



            Lets denote $E_n$ for $n$ consecutive heads.
            Now if we get one more head after $E_{n-1}$, then we have $n$ consecutive heads
            or if it is a tail then again we have to repeat the procedure.



            So for the two scenarios:




            1. $E_{n-1}+1$

            2. $E_{n}{+1}$ ($1$ for a tail)


            So, $E_n=frac12(E_{n-1} +1)+frac12(E_{n-1}+ E_n+ 1)$,
            so $E_n= 2E_{n-1}+2$.



            We have the general recurrence relation. Define $f(n)=E_n+2$ with $f(0)=2$. So,



            begin{align}
            f(n)&=2f(n-1) \
            implies f(n)&=2^{n+1}
            end{align}



            Therefore, $E_n = 2^{n+1}-2 = 2(2^n-1)$



            For $n=5$, it will give us $2(2^5-1)=62$.






            share|cite|improve this answer











            $endgroup$









            • 1




              $begingroup$
              Amazing solution, thank you
              $endgroup$
              – Jaydev
              Jul 24 '17 at 1:30














            14












            14








            14





            $begingroup$

            Lets calculate it for $n$ consecutive tosses the expected number of tosses needed.



            Lets denote $E_n$ for $n$ consecutive heads.
            Now if we get one more head after $E_{n-1}$, then we have $n$ consecutive heads
            or if it is a tail then again we have to repeat the procedure.



            So for the two scenarios:




            1. $E_{n-1}+1$

            2. $E_{n}{+1}$ ($1$ for a tail)


            So, $E_n=frac12(E_{n-1} +1)+frac12(E_{n-1}+ E_n+ 1)$,
            so $E_n= 2E_{n-1}+2$.



            We have the general recurrence relation. Define $f(n)=E_n+2$ with $f(0)=2$. So,



            begin{align}
            f(n)&=2f(n-1) \
            implies f(n)&=2^{n+1}
            end{align}



            Therefore, $E_n = 2^{n+1}-2 = 2(2^n-1)$



            For $n=5$, it will give us $2(2^5-1)=62$.






            share|cite|improve this answer











            $endgroup$



            Lets calculate it for $n$ consecutive tosses the expected number of tosses needed.



            Lets denote $E_n$ for $n$ consecutive heads.
            Now if we get one more head after $E_{n-1}$, then we have $n$ consecutive heads
            or if it is a tail then again we have to repeat the procedure.



            So for the two scenarios:




            1. $E_{n-1}+1$

            2. $E_{n}{+1}$ ($1$ for a tail)


            So, $E_n=frac12(E_{n-1} +1)+frac12(E_{n-1}+ E_n+ 1)$,
            so $E_n= 2E_{n-1}+2$.



            We have the general recurrence relation. Define $f(n)=E_n+2$ with $f(0)=2$. So,



            begin{align}
            f(n)&=2f(n-1) \
            implies f(n)&=2^{n+1}
            end{align}



            Therefore, $E_n = 2^{n+1}-2 = 2(2^n-1)$



            For $n=5$, it will give us $2(2^5-1)=62$.







            share|cite|improve this answer














            share|cite|improve this answer



            share|cite|improve this answer








            edited Dec 2 '15 at 23:36









            karakfa

            2,025811




            2,025811










            answered Jul 30 '13 at 11:16









            ravi pradeepravi pradeep

            14112




            14112








            • 1




              $begingroup$
              Amazing solution, thank you
              $endgroup$
              – Jaydev
              Jul 24 '17 at 1:30














            • 1




              $begingroup$
              Amazing solution, thank you
              $endgroup$
              – Jaydev
              Jul 24 '17 at 1:30








            1




            1




            $begingroup$
            Amazing solution, thank you
            $endgroup$
            – Jaydev
            Jul 24 '17 at 1:30




            $begingroup$
            Amazing solution, thank you
            $endgroup$
            – Jaydev
            Jul 24 '17 at 1:30











            9












            $begingroup$

            This problem is solvable with the next step conditioning method. Let $mu_k$ denote the mean number of tosses until 5 consecutive heads occurs, given that $k$ consecutive heads just occured. Obviously $mu_5=0$. Conditioning on the outcome of the next coin throw:
            $$
            mu_k = 1 + frac{1}{2} mu_{k+1} + frac{1}{2} mu_0
            $$
            Solving the resulting linear system:



            In[28]:= Solve[Table[mu[k] == 1 + 1/2 mu[k + 1] + mu[0]/2, {k, 0, 4}],
            Table[mu[k], {k, 0, 4}]] /. mu[5] -> 0

            Out[28]= {{mu[0] -> 62, mu[1] -> 60, mu[2] -> 56, mu[3] -> 48,
            mu[4] -> 32}}


            Hence the expected number of coin flips $mu_0$ equals 62.






            share|cite|improve this answer









            $endgroup$













            • $begingroup$
              What tool did you use for solving?
              $endgroup$
              – pushpen.paul
              May 11 '15 at 17:19










            • $begingroup$
              @pushpen.paul I used Mathematica
              $endgroup$
              – Sasha
              May 11 '15 at 17:20










            • $begingroup$
              Can you please explain the original equation?
              $endgroup$
              – BOS
              Sep 20 '16 at 15:58










            • $begingroup$
              @BOS Since $mu_k$ is the conditional expectation, consider the next coin toss. Because a new toss was made, we add 1, in the next state, with equal probabilities we either get next head, in which case we gonna get $k+1$ heads, hence $mu_{k+1}$, or the tail, in which case we break the streak of consecutive heads, hence $mu_0$.
              $endgroup$
              – Sasha
              Sep 23 '16 at 3:03






            • 1




              $begingroup$
              @Sasha This is the Markov way of solving it, right? This seems most intuitive to me of all methods presented here.
              $endgroup$
              – user3496060
              Dec 22 '17 at 8:06


















            9












            $begingroup$

            This problem is solvable with the next step conditioning method. Let $mu_k$ denote the mean number of tosses until 5 consecutive heads occurs, given that $k$ consecutive heads just occured. Obviously $mu_5=0$. Conditioning on the outcome of the next coin throw:
            $$
            mu_k = 1 + frac{1}{2} mu_{k+1} + frac{1}{2} mu_0
            $$
            Solving the resulting linear system:



            In[28]:= Solve[Table[mu[k] == 1 + 1/2 mu[k + 1] + mu[0]/2, {k, 0, 4}],
            Table[mu[k], {k, 0, 4}]] /. mu[5] -> 0

            Out[28]= {{mu[0] -> 62, mu[1] -> 60, mu[2] -> 56, mu[3] -> 48,
            mu[4] -> 32}}


            Hence the expected number of coin flips $mu_0$ equals 62.






            share|cite|improve this answer









            $endgroup$













            • $begingroup$
              What tool did you use for solving?
              $endgroup$
              – pushpen.paul
              May 11 '15 at 17:19










            • $begingroup$
              @pushpen.paul I used Mathematica
              $endgroup$
              – Sasha
              May 11 '15 at 17:20










            • $begingroup$
              Can you please explain the original equation?
              $endgroup$
              – BOS
              Sep 20 '16 at 15:58










            • $begingroup$
              @BOS Since $mu_k$ is the conditional expectation, consider the next coin toss. Because a new toss was made, we add 1, in the next state, with equal probabilities we either get next head, in which case we gonna get $k+1$ heads, hence $mu_{k+1}$, or the tail, in which case we break the streak of consecutive heads, hence $mu_0$.
              $endgroup$
              – Sasha
              Sep 23 '16 at 3:03






            • 1




              $begingroup$
              @Sasha This is the Markov way of solving it, right? This seems most intuitive to me of all methods presented here.
              $endgroup$
              – user3496060
              Dec 22 '17 at 8:06
















            9












            9








            9





            $begingroup$

            This problem is solvable with the next step conditioning method. Let $mu_k$ denote the mean number of tosses until 5 consecutive heads occurs, given that $k$ consecutive heads just occured. Obviously $mu_5=0$. Conditioning on the outcome of the next coin throw:
            $$
            mu_k = 1 + frac{1}{2} mu_{k+1} + frac{1}{2} mu_0
            $$
            Solving the resulting linear system:



            In[28]:= Solve[Table[mu[k] == 1 + 1/2 mu[k + 1] + mu[0]/2, {k, 0, 4}],
            Table[mu[k], {k, 0, 4}]] /. mu[5] -> 0

            Out[28]= {{mu[0] -> 62, mu[1] -> 60, mu[2] -> 56, mu[3] -> 48,
            mu[4] -> 32}}


            Hence the expected number of coin flips $mu_0$ equals 62.






            share|cite|improve this answer









            $endgroup$



            This problem is solvable with the next step conditioning method. Let $mu_k$ denote the mean number of tosses until 5 consecutive heads occurs, given that $k$ consecutive heads just occured. Obviously $mu_5=0$. Conditioning on the outcome of the next coin throw:
            $$
            mu_k = 1 + frac{1}{2} mu_{k+1} + frac{1}{2} mu_0
            $$
            Solving the resulting linear system:



            In[28]:= Solve[Table[mu[k] == 1 + 1/2 mu[k + 1] + mu[0]/2, {k, 0, 4}],
            Table[mu[k], {k, 0, 4}]] /. mu[5] -> 0

            Out[28]= {{mu[0] -> 62, mu[1] -> 60, mu[2] -> 56, mu[3] -> 48,
            mu[4] -> 32}}


            Hence the expected number of coin flips $mu_0$ equals 62.







            share|cite|improve this answer












            share|cite|improve this answer



            share|cite|improve this answer










            answered Apr 17 '13 at 3:58









            SashaSasha

            60.9k5110181




            60.9k5110181












            • $begingroup$
              What tool did you use for solving?
              $endgroup$
              – pushpen.paul
              May 11 '15 at 17:19










            • $begingroup$
              @pushpen.paul I used Mathematica
              $endgroup$
              – Sasha
              May 11 '15 at 17:20










            • $begingroup$
              Can you please explain the original equation?
              $endgroup$
              – BOS
              Sep 20 '16 at 15:58










            • $begingroup$
              @BOS Since $mu_k$ is the conditional expectation, consider the next coin toss. Because a new toss was made, we add 1, in the next state, with equal probabilities we either get next head, in which case we gonna get $k+1$ heads, hence $mu_{k+1}$, or the tail, in which case we break the streak of consecutive heads, hence $mu_0$.
              $endgroup$
              – Sasha
              Sep 23 '16 at 3:03






            • 1




              $begingroup$
              @Sasha This is the Markov way of solving it, right? This seems most intuitive to me of all methods presented here.
              $endgroup$
              – user3496060
              Dec 22 '17 at 8:06




















            • $begingroup$
              What tool did you use for solving?
              $endgroup$
              – pushpen.paul
              May 11 '15 at 17:19










            • $begingroup$
              @pushpen.paul I used Mathematica
              $endgroup$
              – Sasha
              May 11 '15 at 17:20










            • $begingroup$
              Can you please explain the original equation?
              $endgroup$
              – BOS
              Sep 20 '16 at 15:58










            • $begingroup$
              @BOS Since $mu_k$ is the conditional expectation, consider the next coin toss. Because a new toss was made, we add 1, in the next state, with equal probabilities we either get next head, in which case we gonna get $k+1$ heads, hence $mu_{k+1}$, or the tail, in which case we break the streak of consecutive heads, hence $mu_0$.
              $endgroup$
              – Sasha
              Sep 23 '16 at 3:03






            • 1




              $begingroup$
              @Sasha This is the Markov way of solving it, right? This seems most intuitive to me of all methods presented here.
              $endgroup$
              – user3496060
              Dec 22 '17 at 8:06


















            $begingroup$
            What tool did you use for solving?
            $endgroup$
            – pushpen.paul
            May 11 '15 at 17:19




            $begingroup$
            What tool did you use for solving?
            $endgroup$
            – pushpen.paul
            May 11 '15 at 17:19












            $begingroup$
            @pushpen.paul I used Mathematica
            $endgroup$
            – Sasha
            May 11 '15 at 17:20




            $begingroup$
            @pushpen.paul I used Mathematica
            $endgroup$
            – Sasha
            May 11 '15 at 17:20












            $begingroup$
            Can you please explain the original equation?
            $endgroup$
            – BOS
            Sep 20 '16 at 15:58




            $begingroup$
            Can you please explain the original equation?
            $endgroup$
            – BOS
            Sep 20 '16 at 15:58












            $begingroup$
            @BOS Since $mu_k$ is the conditional expectation, consider the next coin toss. Because a new toss was made, we add 1, in the next state, with equal probabilities we either get next head, in which case we gonna get $k+1$ heads, hence $mu_{k+1}$, or the tail, in which case we break the streak of consecutive heads, hence $mu_0$.
            $endgroup$
            – Sasha
            Sep 23 '16 at 3:03




            $begingroup$
            @BOS Since $mu_k$ is the conditional expectation, consider the next coin toss. Because a new toss was made, we add 1, in the next state, with equal probabilities we either get next head, in which case we gonna get $k+1$ heads, hence $mu_{k+1}$, or the tail, in which case we break the streak of consecutive heads, hence $mu_0$.
            $endgroup$
            – Sasha
            Sep 23 '16 at 3:03




            1




            1




            $begingroup$
            @Sasha This is the Markov way of solving it, right? This seems most intuitive to me of all methods presented here.
            $endgroup$
            – user3496060
            Dec 22 '17 at 8:06






            $begingroup$
            @Sasha This is the Markov way of solving it, right? This seems most intuitive to me of all methods presented here.
            $endgroup$
            – user3496060
            Dec 22 '17 at 8:06













            2












            $begingroup$

            The question can be generalized to what is the expected number of tosses before we get x heads.Let's call this E(x). We can easily derive a recursive formula for E(x).
            Now, there are a total of two possibilities, first is that we fail to get the xth consecutive heads in xth attempt and second, we succeed. Probability of success is 1/(2^x) and probability of failure is 1-(1/(2^x)).



            Now, if we were to fail to get xth consecutive heads in xth toss (i.e. case 1), the we will have to use a total of (E(x)+1) moves, because one move has been wasted.



            On the other hand if we were to succeed in getting xth consecutive head in xth toss (i.e. case 2), the total moves is E(x-1)+1 , because we now take one move more than that was required to get x-1 consecutive heads.



            So,



            E(x) = P(failure) * (E(x)+1) + P(success) * (E(x-1)+1)

            E(x) = [1-(1/(2^x))] * (E(x)+1) + [1/(2^x)] * (E(x-1)+1)



            Also E(0) = 0 , because expected number of tosses to get 0 heads is zero, duh



            now,



            E(1) = (1-0.5) * (E(1)+1) + (0.5) * (E(0)+1) =>
            E(1) = 2



            E(2) = (1-0.125) * (E(1)+1) + (0.125) * (E(1)+1) =>
            E(2) = 6



            Similarly,



            E(3) = 14



            E(4) = 30



            E(5) = 62






            share|cite|improve this answer











            $endgroup$


















              2












              $begingroup$

              The question can be generalized to what is the expected number of tosses before we get x heads.Let's call this E(x). We can easily derive a recursive formula for E(x).
              Now, there are a total of two possibilities, first is that we fail to get the xth consecutive heads in xth attempt and second, we succeed. Probability of success is 1/(2^x) and probability of failure is 1-(1/(2^x)).



              Now, if we were to fail to get xth consecutive heads in xth toss (i.e. case 1), the we will have to use a total of (E(x)+1) moves, because one move has been wasted.



              On the other hand if we were to succeed in getting xth consecutive head in xth toss (i.e. case 2), the total moves is E(x-1)+1 , because we now take one move more than that was required to get x-1 consecutive heads.



              So,



              E(x) = P(failure) * (E(x)+1) + P(success) * (E(x-1)+1)

              E(x) = [1-(1/(2^x))] * (E(x)+1) + [1/(2^x)] * (E(x-1)+1)



              Also E(0) = 0 , because expected number of tosses to get 0 heads is zero, duh



              now,



              E(1) = (1-0.5) * (E(1)+1) + (0.5) * (E(0)+1) =>
              E(1) = 2



              E(2) = (1-0.125) * (E(1)+1) + (0.125) * (E(1)+1) =>
              E(2) = 6



              Similarly,



              E(3) = 14



              E(4) = 30



              E(5) = 62






              share|cite|improve this answer











              $endgroup$
















                2












                2








                2





                $begingroup$

                The question can be generalized to what is the expected number of tosses before we get x heads.Let's call this E(x). We can easily derive a recursive formula for E(x).
                Now, there are a total of two possibilities, first is that we fail to get the xth consecutive heads in xth attempt and second, we succeed. Probability of success is 1/(2^x) and probability of failure is 1-(1/(2^x)).



                Now, if we were to fail to get xth consecutive heads in xth toss (i.e. case 1), the we will have to use a total of (E(x)+1) moves, because one move has been wasted.



                On the other hand if we were to succeed in getting xth consecutive head in xth toss (i.e. case 2), the total moves is E(x-1)+1 , because we now take one move more than that was required to get x-1 consecutive heads.



                So,



                E(x) = P(failure) * (E(x)+1) + P(success) * (E(x-1)+1)

                E(x) = [1-(1/(2^x))] * (E(x)+1) + [1/(2^x)] * (E(x-1)+1)



                Also E(0) = 0 , because expected number of tosses to get 0 heads is zero, duh



                now,



                E(1) = (1-0.5) * (E(1)+1) + (0.5) * (E(0)+1) =>
                E(1) = 2



                E(2) = (1-0.125) * (E(1)+1) + (0.125) * (E(1)+1) =>
                E(2) = 6



                Similarly,



                E(3) = 14



                E(4) = 30



                E(5) = 62






                share|cite|improve this answer











                $endgroup$



                The question can be generalized to what is the expected number of tosses before we get x heads.Let's call this E(x). We can easily derive a recursive formula for E(x).
                Now, there are a total of two possibilities, first is that we fail to get the xth consecutive heads in xth attempt and second, we succeed. Probability of success is 1/(2^x) and probability of failure is 1-(1/(2^x)).



                Now, if we were to fail to get xth consecutive heads in xth toss (i.e. case 1), the we will have to use a total of (E(x)+1) moves, because one move has been wasted.



                On the other hand if we were to succeed in getting xth consecutive head in xth toss (i.e. case 2), the total moves is E(x-1)+1 , because we now take one move more than that was required to get x-1 consecutive heads.



                So,



                E(x) = P(failure) * (E(x)+1) + P(success) * (E(x-1)+1)

                E(x) = [1-(1/(2^x))] * (E(x)+1) + [1/(2^x)] * (E(x-1)+1)



                Also E(0) = 0 , because expected number of tosses to get 0 heads is zero, duh



                now,



                E(1) = (1-0.5) * (E(1)+1) + (0.5) * (E(0)+1) =>
                E(1) = 2



                E(2) = (1-0.125) * (E(1)+1) + (0.125) * (E(1)+1) =>
                E(2) = 6



                Similarly,



                E(3) = 14



                E(4) = 30



                E(5) = 62







                share|cite|improve this answer














                share|cite|improve this answer



                share|cite|improve this answer








                edited Apr 21 '16 at 16:53









                Community

                1




                1










                answered Sep 9 '14 at 22:23









                Abhimanyu SoodAbhimanyu Sood

                211




                211























                    1












                    $begingroup$

                    I would simplify the problem as follows:



                    Let $e$ = Expected number of flips until $5$ consecutive $H$, i.e., $E[5H]$

                    Let $f$ = Expected number of flips until $5$ consecutive $H$ when we have seen one $H$, i.e., $E[5H|H]$

                    Let $g$ = Expected number of flips until $5$ consecutive $H$ when we have seen two $H$, i.e., $E[5H|2H]$

                    Let $h$ = Expected number of flips until $5$ consecutive $H$ when we have seen three $H$, i.e., $E[5H|3H]$

                    Let $i$ = Expected number of flips until $5$ consecutive $H$ when we have seen four $H$, i.e., $E[5H|4H]$



                    Now Start flipping coin, there is $frac{1}{2}$ probability of getting $H$ or $T$. So if we get $H$ then expected number of flips until 5 consecutive $H$ is $(f+1)$. Alternatively if $T$, we wasted 1 flip and expected number is still $(e+1)$
                    $$
                    e=frac12(e+1)+frac12(f+1);
                    $$



                    We now need $f$ to solve above to get $e$. Now we start with 1 $H$ and seeking 4 more $H$ to get total 5 $H$. Again, there is $frac{1}{2}$ probability of getting $H$ or $T$. So if we get $H$ (total $2H$ so far) then expected number of flips until 5 consecutive $H$ is $(g+1)$. Alternatively if $T$, we wasted this flip and expected number is back to $(e+1)$



                    $$
                    f=frac12(g+1)+frac12(e+1);
                    $$



                    Continuing this way...
                    $$
                    g=frac12(h+1)+frac12(e+1);
                    $$
                    $$
                    h=frac12(i+1)+frac12(e+1);
                    $$



                    Finally, Now we have 4 $H$ and seeking last $H$ to get total 5 $H$. Still, there is $frac{1}{2}$ probability of getting $H$ or $T$. So if we get $H$ (total $5H$) then we need just $1$ flip. Alternatively if $T$ is observed, we wasted this flip and expected number is back to $(e+1)$
                    $$
                    i=frac12(1)+frac12(e+1);
                    $$



                    Solving these equations, $e=62$, $f=60$, $g=56$, $h=48$, $i=32$

                    This solution offers some insight into conditional expectations of number of flips needed till 5 consecutive $H$ given 1, 2, 3 and 4 consecutive $H$.






                    share|cite|improve this answer











                    $endgroup$


















                      1












                      $begingroup$

                      I would simplify the problem as follows:



                      Let $e$ = Expected number of flips until $5$ consecutive $H$, i.e., $E[5H]$

                      Let $f$ = Expected number of flips until $5$ consecutive $H$ when we have seen one $H$, i.e., $E[5H|H]$

                      Let $g$ = Expected number of flips until $5$ consecutive $H$ when we have seen two $H$, i.e., $E[5H|2H]$

                      Let $h$ = Expected number of flips until $5$ consecutive $H$ when we have seen three $H$, i.e., $E[5H|3H]$

                      Let $i$ = Expected number of flips until $5$ consecutive $H$ when we have seen four $H$, i.e., $E[5H|4H]$



                      Now Start flipping coin, there is $frac{1}{2}$ probability of getting $H$ or $T$. So if we get $H$ then expected number of flips until 5 consecutive $H$ is $(f+1)$. Alternatively if $T$, we wasted 1 flip and expected number is still $(e+1)$
                      $$
                      e=frac12(e+1)+frac12(f+1);
                      $$



                      We now need $f$ to solve above to get $e$. Now we start with 1 $H$ and seeking 4 more $H$ to get total 5 $H$. Again, there is $frac{1}{2}$ probability of getting $H$ or $T$. So if we get $H$ (total $2H$ so far) then expected number of flips until 5 consecutive $H$ is $(g+1)$. Alternatively if $T$, we wasted this flip and expected number is back to $(e+1)$



                      $$
                      f=frac12(g+1)+frac12(e+1);
                      $$



                      Continuing this way...
                      $$
                      g=frac12(h+1)+frac12(e+1);
                      $$
                      $$
                      h=frac12(i+1)+frac12(e+1);
                      $$



                      Finally, Now we have 4 $H$ and seeking last $H$ to get total 5 $H$. Still, there is $frac{1}{2}$ probability of getting $H$ or $T$. So if we get $H$ (total $5H$) then we need just $1$ flip. Alternatively if $T$ is observed, we wasted this flip and expected number is back to $(e+1)$
                      $$
                      i=frac12(1)+frac12(e+1);
                      $$



                      Solving these equations, $e=62$, $f=60$, $g=56$, $h=48$, $i=32$

                      This solution offers some insight into conditional expectations of number of flips needed till 5 consecutive $H$ given 1, 2, 3 and 4 consecutive $H$.






                      share|cite|improve this answer











                      $endgroup$
















                        1












                        1








                        1





                        $begingroup$

                        I would simplify the problem as follows:



                        Let $e$ = Expected number of flips until $5$ consecutive $H$, i.e., $E[5H]$

                        Let $f$ = Expected number of flips until $5$ consecutive $H$ when we have seen one $H$, i.e., $E[5H|H]$

                        Let $g$ = Expected number of flips until $5$ consecutive $H$ when we have seen two $H$, i.e., $E[5H|2H]$

                        Let $h$ = Expected number of flips until $5$ consecutive $H$ when we have seen three $H$, i.e., $E[5H|3H]$

                        Let $i$ = Expected number of flips until $5$ consecutive $H$ when we have seen four $H$, i.e., $E[5H|4H]$



                        Now Start flipping coin, there is $frac{1}{2}$ probability of getting $H$ or $T$. So if we get $H$ then expected number of flips until 5 consecutive $H$ is $(f+1)$. Alternatively if $T$, we wasted 1 flip and expected number is still $(e+1)$
                        $$
                        e=frac12(e+1)+frac12(f+1);
                        $$



                        We now need $f$ to solve above to get $e$. Now we start with 1 $H$ and seeking 4 more $H$ to get total 5 $H$. Again, there is $frac{1}{2}$ probability of getting $H$ or $T$. So if we get $H$ (total $2H$ so far) then expected number of flips until 5 consecutive $H$ is $(g+1)$. Alternatively if $T$, we wasted this flip and expected number is back to $(e+1)$



                        $$
                        f=frac12(g+1)+frac12(e+1);
                        $$



                        Continuing this way...
                        $$
                        g=frac12(h+1)+frac12(e+1);
                        $$
                        $$
                        h=frac12(i+1)+frac12(e+1);
                        $$



                        Finally, Now we have 4 $H$ and seeking last $H$ to get total 5 $H$. Still, there is $frac{1}{2}$ probability of getting $H$ or $T$. So if we get $H$ (total $5H$) then we need just $1$ flip. Alternatively if $T$ is observed, we wasted this flip and expected number is back to $(e+1)$
                        $$
                        i=frac12(1)+frac12(e+1);
                        $$



                        Solving these equations, $e=62$, $f=60$, $g=56$, $h=48$, $i=32$

                        This solution offers some insight into conditional expectations of number of flips needed till 5 consecutive $H$ given 1, 2, 3 and 4 consecutive $H$.






                        share|cite|improve this answer











                        $endgroup$



                        I would simplify the problem as follows:



                        Let $e$ = Expected number of flips until $5$ consecutive $H$, i.e., $E[5H]$

                        Let $f$ = Expected number of flips until $5$ consecutive $H$ when we have seen one $H$, i.e., $E[5H|H]$

                        Let $g$ = Expected number of flips until $5$ consecutive $H$ when we have seen two $H$, i.e., $E[5H|2H]$

                        Let $h$ = Expected number of flips until $5$ consecutive $H$ when we have seen three $H$, i.e., $E[5H|3H]$

                        Let $i$ = Expected number of flips until $5$ consecutive $H$ when we have seen four $H$, i.e., $E[5H|4H]$



                        Now Start flipping coin, there is $frac{1}{2}$ probability of getting $H$ or $T$. So if we get $H$ then expected number of flips until 5 consecutive $H$ is $(f+1)$. Alternatively if $T$, we wasted 1 flip and expected number is still $(e+1)$
                        $$
                        e=frac12(e+1)+frac12(f+1);
                        $$



                        We now need $f$ to solve above to get $e$. Now we start with 1 $H$ and seeking 4 more $H$ to get total 5 $H$. Again, there is $frac{1}{2}$ probability of getting $H$ or $T$. So if we get $H$ (total $2H$ so far) then expected number of flips until 5 consecutive $H$ is $(g+1)$. Alternatively if $T$, we wasted this flip and expected number is back to $(e+1)$



                        $$
                        f=frac12(g+1)+frac12(e+1);
                        $$



                        Continuing this way...
                        $$
                        g=frac12(h+1)+frac12(e+1);
                        $$
                        $$
                        h=frac12(i+1)+frac12(e+1);
                        $$



                        Finally, Now we have 4 $H$ and seeking last $H$ to get total 5 $H$. Still, there is $frac{1}{2}$ probability of getting $H$ or $T$. So if we get $H$ (total $5H$) then we need just $1$ flip. Alternatively if $T$ is observed, we wasted this flip and expected number is back to $(e+1)$
                        $$
                        i=frac12(1)+frac12(e+1);
                        $$



                        Solving these equations, $e=62$, $f=60$, $g=56$, $h=48$, $i=32$

                        This solution offers some insight into conditional expectations of number of flips needed till 5 consecutive $H$ given 1, 2, 3 and 4 consecutive $H$.







                        share|cite|improve this answer














                        share|cite|improve this answer



                        share|cite|improve this answer








                        edited May 2 '16 at 19:59

























                        answered May 2 '16 at 19:40









                        RahulRahul

                        17410




                        17410























                            0












                            $begingroup$

                            Expectation for getting n consecutive heads is : 2*(2^n-1). Thus for 5 heads it is = 62.






                            share|cite|improve this answer









                            $endgroup$









                            • 6




                              $begingroup$
                              How did you get the formula?
                              $endgroup$
                              – pushpen.paul
                              May 11 '15 at 18:25
















                            0












                            $begingroup$

                            Expectation for getting n consecutive heads is : 2*(2^n-1). Thus for 5 heads it is = 62.






                            share|cite|improve this answer









                            $endgroup$









                            • 6




                              $begingroup$
                              How did you get the formula?
                              $endgroup$
                              – pushpen.paul
                              May 11 '15 at 18:25














                            0












                            0








                            0





                            $begingroup$

                            Expectation for getting n consecutive heads is : 2*(2^n-1). Thus for 5 heads it is = 62.






                            share|cite|improve this answer









                            $endgroup$



                            Expectation for getting n consecutive heads is : 2*(2^n-1). Thus for 5 heads it is = 62.







                            share|cite|improve this answer












                            share|cite|improve this answer



                            share|cite|improve this answer










                            answered Nov 20 '14 at 7:40









                            Subhabrata DebnathSubhabrata Debnath

                            11




                            11








                            • 6




                              $begingroup$
                              How did you get the formula?
                              $endgroup$
                              – pushpen.paul
                              May 11 '15 at 18:25














                            • 6




                              $begingroup$
                              How did you get the formula?
                              $endgroup$
                              – pushpen.paul
                              May 11 '15 at 18:25








                            6




                            6




                            $begingroup$
                            How did you get the formula?
                            $endgroup$
                            – pushpen.paul
                            May 11 '15 at 18:25




                            $begingroup$
                            How did you get the formula?
                            $endgroup$
                            – pushpen.paul
                            May 11 '15 at 18:25











                            0












                            $begingroup$

                            No one else seems to have suggested the following approach. Suppose we keep flipping a coin until we get five heads in a row. Define a "run" as either five consecutive heads or a tails flip plus the preceding streak of heads flips. (A run could be a single tails flip.) The number of coin flips is equal to the number of runs with at least four heads ($R_{4+}$), plus the number of runs with at least three heads ($R_{3+}$), and so on down to the number of runs with at least zero heads ($R_{0+}$). We can see this by expanding the terms:



                            $R_{0+}$ = # runs with 0 heads + # runs with 1 head + ... + # runs with 5 heads



                            $R_{1+}$ = # runs with 1 head + # runs with 2 heads + ... + # runs with 5 heads



                            ...



                            $R_{4+}$ = # runs with 4 heads + # runs with 5 heads



                            # flips = # flips in runs with 0 heads + # flips in runs with 1 head + ... + # flips in runs with 5 heads



                            # flips in runs with 0 heads = # runs with 0 heads



                            # flips in runs with 1 head = 2 x # runs with 1 head



                            ...



                            # flips in runs with 4 heads = 5 x # runs with 4 heads



                            # flips in runs with 5 heads = 5 x # runs with 5 heads



                            By linearity of expectation, the expected number of coin flips is $E(R_{0+}) + E(R_{1+}) + ldots + E(R_{4+})$. $E(R_{4+})$ is $2 E(R_{5+}) = 2$, because one half of the time we flip at least four heads in a row, we go on to flip five heads in a row, i.e. the following coin flip is heads. In other words, in expectation, it takes two runs that start with four heads to achieve one run of five heads. Likewise, $E(R_{3+}) = 2 E(R_{4+}) = 4$, $E(R_{2+}) = 2 E(R_{3+}) = 8$, $E(R_{1+}) = 2 E(R_{2+}) = 16$, and $E(R_{0+}) = 2 E(R_{1+}) = 32$. The expected number of coin flips is $32 + 16 + 8 + 4 + 2 = 62$.



                            More generally, given a biased coin that comes up heads $p$ portion of the time, the expected number of flips to get $n$ heads in a row is $frac{1}{p} + frac{1}{p^2} + ldots + frac{1}{p^n} = frac{1 - p^n}{p^n(1 - p)}$.






                            share|cite|improve this answer









                            $endgroup$


















                              0












                              $begingroup$

                              No one else seems to have suggested the following approach. Suppose we keep flipping a coin until we get five heads in a row. Define a "run" as either five consecutive heads or a tails flip plus the preceding streak of heads flips. (A run could be a single tails flip.) The number of coin flips is equal to the number of runs with at least four heads ($R_{4+}$), plus the number of runs with at least three heads ($R_{3+}$), and so on down to the number of runs with at least zero heads ($R_{0+}$). We can see this by expanding the terms:



                              $R_{0+}$ = # runs with 0 heads + # runs with 1 head + ... + # runs with 5 heads



                              $R_{1+}$ = # runs with 1 head + # runs with 2 heads + ... + # runs with 5 heads



                              ...



                              $R_{4+}$ = # runs with 4 heads + # runs with 5 heads



                              # flips = # flips in runs with 0 heads + # flips in runs with 1 head + ... + # flips in runs with 5 heads



                              # flips in runs with 0 heads = # runs with 0 heads



                              # flips in runs with 1 head = 2 x # runs with 1 head



                              ...



                              # flips in runs with 4 heads = 5 x # runs with 4 heads



                              # flips in runs with 5 heads = 5 x # runs with 5 heads



                              By linearity of expectation, the expected number of coin flips is $E(R_{0+}) + E(R_{1+}) + ldots + E(R_{4+})$. $E(R_{4+})$ is $2 E(R_{5+}) = 2$, because one half of the time we flip at least four heads in a row, we go on to flip five heads in a row, i.e. the following coin flip is heads. In other words, in expectation, it takes two runs that start with four heads to achieve one run of five heads. Likewise, $E(R_{3+}) = 2 E(R_{4+}) = 4$, $E(R_{2+}) = 2 E(R_{3+}) = 8$, $E(R_{1+}) = 2 E(R_{2+}) = 16$, and $E(R_{0+}) = 2 E(R_{1+}) = 32$. The expected number of coin flips is $32 + 16 + 8 + 4 + 2 = 62$.



                              More generally, given a biased coin that comes up heads $p$ portion of the time, the expected number of flips to get $n$ heads in a row is $frac{1}{p} + frac{1}{p^2} + ldots + frac{1}{p^n} = frac{1 - p^n}{p^n(1 - p)}$.






                              share|cite|improve this answer









                              $endgroup$
















                                0












                                0








                                0





                                $begingroup$

                                No one else seems to have suggested the following approach. Suppose we keep flipping a coin until we get five heads in a row. Define a "run" as either five consecutive heads or a tails flip plus the preceding streak of heads flips. (A run could be a single tails flip.) The number of coin flips is equal to the number of runs with at least four heads ($R_{4+}$), plus the number of runs with at least three heads ($R_{3+}$), and so on down to the number of runs with at least zero heads ($R_{0+}$). We can see this by expanding the terms:



                                $R_{0+}$ = # runs with 0 heads + # runs with 1 head + ... + # runs with 5 heads



                                $R_{1+}$ = # runs with 1 head + # runs with 2 heads + ... + # runs with 5 heads



                                ...



                                $R_{4+}$ = # runs with 4 heads + # runs with 5 heads



                                # flips = # flips in runs with 0 heads + # flips in runs with 1 head + ... + # flips in runs with 5 heads



                                # flips in runs with 0 heads = # runs with 0 heads



                                # flips in runs with 1 head = 2 x # runs with 1 head



                                ...



                                # flips in runs with 4 heads = 5 x # runs with 4 heads



                                # flips in runs with 5 heads = 5 x # runs with 5 heads



                                By linearity of expectation, the expected number of coin flips is $E(R_{0+}) + E(R_{1+}) + ldots + E(R_{4+})$. $E(R_{4+})$ is $2 E(R_{5+}) = 2$, because one half of the time we flip at least four heads in a row, we go on to flip five heads in a row, i.e. the following coin flip is heads. In other words, in expectation, it takes two runs that start with four heads to achieve one run of five heads. Likewise, $E(R_{3+}) = 2 E(R_{4+}) = 4$, $E(R_{2+}) = 2 E(R_{3+}) = 8$, $E(R_{1+}) = 2 E(R_{2+}) = 16$, and $E(R_{0+}) = 2 E(R_{1+}) = 32$. The expected number of coin flips is $32 + 16 + 8 + 4 + 2 = 62$.



                                More generally, given a biased coin that comes up heads $p$ portion of the time, the expected number of flips to get $n$ heads in a row is $frac{1}{p} + frac{1}{p^2} + ldots + frac{1}{p^n} = frac{1 - p^n}{p^n(1 - p)}$.






                                share|cite|improve this answer









                                $endgroup$



                                No one else seems to have suggested the following approach. Suppose we keep flipping a coin until we get five heads in a row. Define a "run" as either five consecutive heads or a tails flip plus the preceding streak of heads flips. (A run could be a single tails flip.) The number of coin flips is equal to the number of runs with at least four heads ($R_{4+}$), plus the number of runs with at least three heads ($R_{3+}$), and so on down to the number of runs with at least zero heads ($R_{0+}$). We can see this by expanding the terms:



                                $R_{0+}$ = # runs with 0 heads + # runs with 1 head + ... + # runs with 5 heads



                                $R_{1+}$ = # runs with 1 head + # runs with 2 heads + ... + # runs with 5 heads



                                ...



                                $R_{4+}$ = # runs with 4 heads + # runs with 5 heads



                                # flips = # flips in runs with 0 heads + # flips in runs with 1 head + ... + # flips in runs with 5 heads



                                # flips in runs with 0 heads = # runs with 0 heads



                                # flips in runs with 1 head = 2 x # runs with 1 head



                                ...



                                # flips in runs with 4 heads = 5 x # runs with 4 heads



                                # flips in runs with 5 heads = 5 x # runs with 5 heads



                                By linearity of expectation, the expected number of coin flips is $E(R_{0+}) + E(R_{1+}) + ldots + E(R_{4+})$. $E(R_{4+})$ is $2 E(R_{5+}) = 2$, because one half of the time we flip at least four heads in a row, we go on to flip five heads in a row, i.e. the following coin flip is heads. In other words, in expectation, it takes two runs that start with four heads to achieve one run of five heads. Likewise, $E(R_{3+}) = 2 E(R_{4+}) = 4$, $E(R_{2+}) = 2 E(R_{3+}) = 8$, $E(R_{1+}) = 2 E(R_{2+}) = 16$, and $E(R_{0+}) = 2 E(R_{1+}) = 32$. The expected number of coin flips is $32 + 16 + 8 + 4 + 2 = 62$.



                                More generally, given a biased coin that comes up heads $p$ portion of the time, the expected number of flips to get $n$ heads in a row is $frac{1}{p} + frac{1}{p^2} + ldots + frac{1}{p^n} = frac{1 - p^n}{p^n(1 - p)}$.







                                share|cite|improve this answer












                                share|cite|improve this answer



                                share|cite|improve this answer










                                answered Jun 21 '16 at 17:33









                                btrekkiebtrekkie

                                1




                                1























                                    0












                                    $begingroup$

                                    We can solve this without equations. Ask the following (auxiliary) question: how many flips you need to get either $N$ heads or $N$ tails. Then to get $N$ heads only you need twice as many flips. Start with the question of how many flips you need to get either $H$ or $T$. The answer is
                                    $$x = frac12 (1) + frac12 (1) = 1.$$
                                    The reason is that there is $frac12$ probability to get $H$, and after $1$ flip you are done. The same for $T$. To get only one $H$ you then need two flips. OK. Now we ask what it takes to get $HH$ or $TT$. The result is
                                    $$x=frac12(1+2) + frac12(1+2).$$ The number $2$ appears because, say you flip $H$ first, then you need on average $2$ flips to get another $H$, as we learned earlier. The same for $T$. So you need $3$ flips to get $TT$ or $HH$, and you need $6$ flips to get $HH$ only. And so on. You need $frac12 (1+6) + frac12(1+6) = 7$ flips to get either $HHH$ or $TTT$, and $14$ to get $HHH$ only. If you need $HHHH$ or $TTTT$, then flip $frac12(1+14) + frac12(1+14) = 15$ times, or $30$ times to get just $HHHH$. The sequence is $1, 3, 7, 15, ldots$ to get either heads or tails. The formula is easy to extract: you need $2^N-1$ flips to get either $N$ heads or $N$ tails, or $2^{N+1}-2$ to get $N$ heads only. If $N=5$ we get the answer: $62$.






                                    share|cite|improve this answer











                                    $endgroup$


















                                      0












                                      $begingroup$

                                      We can solve this without equations. Ask the following (auxiliary) question: how many flips you need to get either $N$ heads or $N$ tails. Then to get $N$ heads only you need twice as many flips. Start with the question of how many flips you need to get either $H$ or $T$. The answer is
                                      $$x = frac12 (1) + frac12 (1) = 1.$$
                                      The reason is that there is $frac12$ probability to get $H$, and after $1$ flip you are done. The same for $T$. To get only one $H$ you then need two flips. OK. Now we ask what it takes to get $HH$ or $TT$. The result is
                                      $$x=frac12(1+2) + frac12(1+2).$$ The number $2$ appears because, say you flip $H$ first, then you need on average $2$ flips to get another $H$, as we learned earlier. The same for $T$. So you need $3$ flips to get $TT$ or $HH$, and you need $6$ flips to get $HH$ only. And so on. You need $frac12 (1+6) + frac12(1+6) = 7$ flips to get either $HHH$ or $TTT$, and $14$ to get $HHH$ only. If you need $HHHH$ or $TTTT$, then flip $frac12(1+14) + frac12(1+14) = 15$ times, or $30$ times to get just $HHHH$. The sequence is $1, 3, 7, 15, ldots$ to get either heads or tails. The formula is easy to extract: you need $2^N-1$ flips to get either $N$ heads or $N$ tails, or $2^{N+1}-2$ to get $N$ heads only. If $N=5$ we get the answer: $62$.






                                      share|cite|improve this answer











                                      $endgroup$
















                                        0












                                        0








                                        0





                                        $begingroup$

                                        We can solve this without equations. Ask the following (auxiliary) question: how many flips you need to get either $N$ heads or $N$ tails. Then to get $N$ heads only you need twice as many flips. Start with the question of how many flips you need to get either $H$ or $T$. The answer is
                                        $$x = frac12 (1) + frac12 (1) = 1.$$
                                        The reason is that there is $frac12$ probability to get $H$, and after $1$ flip you are done. The same for $T$. To get only one $H$ you then need two flips. OK. Now we ask what it takes to get $HH$ or $TT$. The result is
                                        $$x=frac12(1+2) + frac12(1+2).$$ The number $2$ appears because, say you flip $H$ first, then you need on average $2$ flips to get another $H$, as we learned earlier. The same for $T$. So you need $3$ flips to get $TT$ or $HH$, and you need $6$ flips to get $HH$ only. And so on. You need $frac12 (1+6) + frac12(1+6) = 7$ flips to get either $HHH$ or $TTT$, and $14$ to get $HHH$ only. If you need $HHHH$ or $TTTT$, then flip $frac12(1+14) + frac12(1+14) = 15$ times, or $30$ times to get just $HHHH$. The sequence is $1, 3, 7, 15, ldots$ to get either heads or tails. The formula is easy to extract: you need $2^N-1$ flips to get either $N$ heads or $N$ tails, or $2^{N+1}-2$ to get $N$ heads only. If $N=5$ we get the answer: $62$.






                                        share|cite|improve this answer











                                        $endgroup$



                                        We can solve this without equations. Ask the following (auxiliary) question: how many flips you need to get either $N$ heads or $N$ tails. Then to get $N$ heads only you need twice as many flips. Start with the question of how many flips you need to get either $H$ or $T$. The answer is
                                        $$x = frac12 (1) + frac12 (1) = 1.$$
                                        The reason is that there is $frac12$ probability to get $H$, and after $1$ flip you are done. The same for $T$. To get only one $H$ you then need two flips. OK. Now we ask what it takes to get $HH$ or $TT$. The result is
                                        $$x=frac12(1+2) + frac12(1+2).$$ The number $2$ appears because, say you flip $H$ first, then you need on average $2$ flips to get another $H$, as we learned earlier. The same for $T$. So you need $3$ flips to get $TT$ or $HH$, and you need $6$ flips to get $HH$ only. And so on. You need $frac12 (1+6) + frac12(1+6) = 7$ flips to get either $HHH$ or $TTT$, and $14$ to get $HHH$ only. If you need $HHHH$ or $TTTT$, then flip $frac12(1+14) + frac12(1+14) = 15$ times, or $30$ times to get just $HHHH$. The sequence is $1, 3, 7, 15, ldots$ to get either heads or tails. The formula is easy to extract: you need $2^N-1$ flips to get either $N$ heads or $N$ tails, or $2^{N+1}-2$ to get $N$ heads only. If $N=5$ we get the answer: $62$.







                                        share|cite|improve this answer














                                        share|cite|improve this answer



                                        share|cite|improve this answer








                                        edited Dec 16 '17 at 22:35









                                        Siong Thye Goh

                                        103k1468119




                                        103k1468119










                                        answered Dec 16 '17 at 22:13









                                        Jaro FabianJaro Fabian

                                        1




                                        1























                                            0












                                            $begingroup$

                                            Use Markov chains. The nice part of Markov Chains is that they can be applied to a huge class of similar problems with relatively little thought (it's almost formulaic in application). It's also the most intuitive way to handle these problems.



                                            (in Matlab code notation below)



                                            %% setup full transition matrix with states from zero heads to 5 heads



                                            T = $[ones(5,1)*.5,eye(5)*.5];$



                                            $T = [T;zeros(1,6)]$



                                            %%Take subset "Q" comprised of just transient states (5 heads is absorbing state)
                                            $Q = T(1:end-1,1:end-1);$



                                            $M = inv(eye(5)-Q)$



                                            absorbing Markov Chain has a similar example as this question BTW...



                                            ans =



                                            62
                                            60
                                            56
                                            48
                                            32


                                            Where each row is the expected number of steps before being absorbed when starting in that transient state (0 through 4 heads, top to bottom).






                                            share|cite|improve this answer









                                            $endgroup$


















                                              0












                                              $begingroup$

                                              Use Markov chains. The nice part of Markov Chains is that they can be applied to a huge class of similar problems with relatively little thought (it's almost formulaic in application). It's also the most intuitive way to handle these problems.



                                              (in Matlab code notation below)



                                              %% setup full transition matrix with states from zero heads to 5 heads



                                              T = $[ones(5,1)*.5,eye(5)*.5];$



                                              $T = [T;zeros(1,6)]$



                                              %%Take subset "Q" comprised of just transient states (5 heads is absorbing state)
                                              $Q = T(1:end-1,1:end-1);$



                                              $M = inv(eye(5)-Q)$



                                              absorbing Markov Chain has a similar example as this question BTW...



                                              ans =



                                              62
                                              60
                                              56
                                              48
                                              32


                                              Where each row is the expected number of steps before being absorbed when starting in that transient state (0 through 4 heads, top to bottom).






                                              share|cite|improve this answer









                                              $endgroup$
















                                                0












                                                0








                                                0





                                                $begingroup$

                                                Use Markov chains. The nice part of Markov Chains is that they can be applied to a huge class of similar problems with relatively little thought (it's almost formulaic in application). It's also the most intuitive way to handle these problems.



                                                (in Matlab code notation below)



                                                %% setup full transition matrix with states from zero heads to 5 heads



                                                T = $[ones(5,1)*.5,eye(5)*.5];$



                                                $T = [T;zeros(1,6)]$



                                                %%Take subset "Q" comprised of just transient states (5 heads is absorbing state)
                                                $Q = T(1:end-1,1:end-1);$



                                                $M = inv(eye(5)-Q)$



                                                absorbing Markov Chain has a similar example as this question BTW...



                                                ans =



                                                62
                                                60
                                                56
                                                48
                                                32


                                                Where each row is the expected number of steps before being absorbed when starting in that transient state (0 through 4 heads, top to bottom).






                                                share|cite|improve this answer









                                                $endgroup$



                                                Use Markov chains. The nice part of Markov Chains is that they can be applied to a huge class of similar problems with relatively little thought (it's almost formulaic in application). It's also the most intuitive way to handle these problems.



                                                (in Matlab code notation below)



                                                %% setup full transition matrix with states from zero heads to 5 heads



                                                T = $[ones(5,1)*.5,eye(5)*.5];$



                                                $T = [T;zeros(1,6)]$



                                                %%Take subset "Q" comprised of just transient states (5 heads is absorbing state)
                                                $Q = T(1:end-1,1:end-1);$



                                                $M = inv(eye(5)-Q)$



                                                absorbing Markov Chain has a similar example as this question BTW...



                                                ans =



                                                62
                                                60
                                                56
                                                48
                                                32


                                                Where each row is the expected number of steps before being absorbed when starting in that transient state (0 through 4 heads, top to bottom).







                                                share|cite|improve this answer












                                                share|cite|improve this answer



                                                share|cite|improve this answer










                                                answered Jan 14 '18 at 5:47









                                                user3496060user3496060

                                                1184




                                                1184






























                                                    draft saved

                                                    draft discarded




















































                                                    Thanks for contributing an answer to Mathematics Stack Exchange!


                                                    • Please be sure to answer the question. Provide details and share your research!

                                                    But avoid



                                                    • Asking for help, clarification, or responding to other answers.

                                                    • Making statements based on opinion; back them up with references or personal experience.


                                                    Use MathJax to format equations. MathJax reference.


                                                    To learn more, see our tips on writing great answers.




                                                    draft saved


                                                    draft discarded














                                                    StackExchange.ready(
                                                    function () {
                                                    StackExchange.openid.initPostLogin('.new-post-login', 'https%3a%2f%2fmath.stackexchange.com%2fquestions%2f364038%2fexpected-number-of-coin-tosses-to-get-five-consecutive-heads%23new-answer', 'question_page');
                                                    }
                                                    );

                                                    Post as a guest















                                                    Required, but never shown





















































                                                    Required, but never shown














                                                    Required, but never shown












                                                    Required, but never shown







                                                    Required, but never shown

































                                                    Required, but never shown














                                                    Required, but never shown












                                                    Required, but never shown







                                                    Required, but never shown







                                                    Popular posts from this blog

                                                    Human spaceflight

                                                    Can not write log (Is /dev/pts mounted?) - openpty in Ubuntu-on-Windows?

                                                    File:DeusFollowingSea.jpg